Teorija_brojeva_radna_verzija.pdf

59
T E O R I J A B R O J E V A 2+2=? Skripta sa predavanja (2004/05) -radna verzija- Pripremio: Mirnes Smajilović Univerzitet u Tuzli Prirodno – matematički fakultet MATEMATIKA

Transcript of Teorija_brojeva_radna_verzija.pdf

Page 1: Teorija_brojeva_radna_verzija.pdf

T E O R I J A B R O J E V A

2+2=?

Skripta sa predavanja (2004/05) -radna verzija-

Pripremio: Mirnes Smajilović

Univerzitet u Tuzli Prirodno – matematički fakultet

MATEMATIKA

Page 2: Teorija_brojeva_radna_verzija.pdf

2

Sadržaj

Sadržaj..........................................................................................................................................2 Uvod.............................................................................................................................................3 I DJELJIVOST CIJELIH BROJEVA.........................................................................................4 II PROSTI BROJEVI..................................................................................................................6 Eratostenovo sito..........................................................................................................................8 III EUKLIDOV ALGORITAM..................................................................................................9 IV FUNDAMENTALNI TEOREM ARITMETIKE................................................................13 V LINEARNE DIOFANTOVE JEDNAČINE.........................................................................17 VI KONGRUENCIJE...............................................................................................................24 VII LINEARNE KONGRUENCIJE.........................................................................................32 VIII KINESKA TEOREMA O OSTACIMA (KTO) ...............................................................35 IX SISTEMI LINEARNIH KONGRUENCIJA .......................................................................38 X TESTOVI DJELJIVOSTI .....................................................................................................40 Test djeljivosti stepenima sa bazom 2........................................................................................40 Test djeljivosti stepenima sa bazom 5........................................................................................40 Test djeljivosti brojevima 3 i 9 ..................................................................................................40 Test djeljivosti brojem 11 ..........................................................................................................41 Test djeljivosti brojevima 7, 11 i 13 ..........................................................................................41 XI WILSONOVA TEOREMA. MALA FERMATOVA TEOREMA.....................................42 XII EULEROVA TEOREMA ..................................................................................................47 Nalaženje inverza broja a po modulu m pomoću Eulerove teoreme..........................................50 XIII PRIMITIVNI KORIJEN ....................................................................................................52 XIV LAGRANGEOVA TEOREMA ........................................................................................56 Dodatak: pisemni dijelovi ispita ................................................................................................59

Page 3: Teorija_brojeva_radna_verzija.pdf

3

Uvod Teorija brojeva je matematička oblast koja je interesovala matematičare hiljadama godina. Kao prvo, javilo se pitanje reprezentacije (predstavljanja) brojeva. Različiti narodi su imali različite načine reprezentacija, npr. stari Babilonci su kao bazu brojnog sistema koristili 60, Maje broj 20, dok je naš sistem reprezentacije prvi put korišten u Indiji prije oko 600 godina. Danas se puno koristi u kompjuterskim naukama binarni sistem. Teorija brojeva se uglavnom bavi proučavanjem cijelih brojeva (Ζ). Skup Ζ je zatvoren u odnosu na operacije sabiranja, oduzimanja i množenja, ali nije zatvoren u odnosu na operaciju dijeljenja i baš pitanje djeljivosti leži u osnovi velikog dijela teorije brojeva. Još prije 2500 godina stari su Grci u Pitagorinoj školi (Pitagorejci) znali za razliku između prostih i složenih brojeva. Dokaz da skup prostih brojeva nije konačan nalazi se još kod Euklida (u njegovom djelu „Elementi“). Slijedeće, vrlo jednostavno pitanje ni do danas nije odgovoreno: Da li postoji formula kojom možemo zadati proste brojeve? Ovo je ilustracija činjenice da u teoriji brojeva postoji mnogo hipoteza koje imaju izuzetno jednostavnu formulaciju a koje ni do danas nisu dokazane. Čak je veliki francuski matematičar P. Fermat (Ferma) (XVII st.) pogrešno pretpostavio da su svi brojevi oblika 22 1,= + ∈

n

nF n N prosti pošto je dokazao za 0,1,2,3,4.=n (Ovo je primjer pogrešnog zaključivanja pomoću nepotpune indukcije). Kasnije je veliki švajcarski matematičar Leonard Euler (XVIII st.) dokazao da je

( ) ( )5 2 22 325 2 1 2 1 256 256 1 6700417 641= + = + = ⋅ + = ⋅F

djeljiv sa 641 pa nije prost broj. Na kraju, napomenimo da je teorija brojeva važna grana matematike, a posebno u zadnjih 100. godina kada su razvijene algebarske i analitičke metode u teoriji brojeva. Za primjenu tih metoda potrebno je veliko znanje analize i algebre. Mi ćemo usvojiti osnovne pojmove i teoreme u elementarnoj teoriji brojeva koji su potrebni za usavršavanje.

Page 4: Teorija_brojeva_radna_verzija.pdf

4

I DJELJIVOST CIJELIH BROJEVA Definicija 1.1 (o dijeljenju u Z ) Za broj ∈a Z kažemo da je djeljiv brojem ∈b Z i pišemo Ma b ili |b a (čitamo na jedan od načina: a je djeljiv sa b, a je sadržalac broja b, a je umnožak ili višekratnik broja b, b dijeli a) akko ( )∃ ∈ = ⋅q a b qZ . Teorema 1.1. a) ( )| |= ∀ ∈a b a bc c Z b) | | |∧ ⇒a b b c a c c) ( ) ( )| | | ,∧ ⇒ + ∀ ∈a b a c a mb nc m n Z d) | |∧ ⇒ = ±a b b a a b e) | 0 0∧ > ∧ > ⇒ ≤a b a b a b Dokaz

a) ( ) ( ).1.1. .1.1.

| |⇒ ∃ ∈ = ⋅ ⇒ = ⇒def def

a b q b a q bc a cq a bcZ

b) ( ) ( )| | , |∧ ⇒ ∃ ∈ = ∧ = ⇒ = ⇒a b b c q s b aq c bs c a qs a cZ c) ( ) ( )| | , |∧ ⇒ ∃ ∈ = ∧ = ⇒ + = + = + ⇒ +a b a c q s b aq c as bm cn maq nas a mq ns a bm cnZ d) ( ) ( ) ( )| | | | 1 0∧ ⇒ ∃ ∈ = ∧ ⇒ ⇒ ∃ ∈ = ⇒ − =a b b a q b aq b a qa a s a sqa a sqZ Z

Imamo dva slučaja: (i) 0 0= ⇒ =a b (ii) ( ) ( ) ( )( )1 0 1 1 1 1− = ⇒ = ∧ = ∨ = − ∧ = − ⇒ = ∨ = −sq q s q s b a b a

e) ( ) ( )| 0 0 0 1∧ > ∧ > ⇒ ∃ < ∈ = ⇒ ≥ ⇒ ≥a b a b q b aq q b aZ . Teorema 1.2. Neka su 1 2, ,..., ∈ka a a Z i 1 2 ... 0+ + + =ka a a . Ako su svi sabirci osim jednog djeljivi brojem ∈p Z , onda je i taj sabirak djeljiv brojem ∈p Z . Dokaz

Bez umanjenja opštosti možemo pretpostaviti da je (*) 1 2 1| ... −+ + + kp a a a Na osnovu definicije 1.1. slijedi ( ) ( )1 2 1, ,..., 1, 2,..., 1k i iq q q a q p i k−∃ ∈ = = −Z

Iz jednakosti 1 2 ... 0+ + + =ka a a i (*) slijedi

( )1 1

1 2 11 1

...k k

k k i ii i

a a a a p q p q− −

−= =

⎛ ⎞= − + + + = − = −⎜ ⎟⎝ ⎠

∑ ∑

a ovo znači da je | kp a , što je i trebalo dokazati.

Page 5: Teorija_brojeva_radna_verzija.pdf

5

Teorema 1.3. (algoritam dijeljenja) Neka su ∈a Z i ∈b N . Tada postoje jedinstveni brojevi , ∈q r Z takvi da vrijedi:

0= + ≤ <a bq r r b Dokaz

Posmatrajmo skup { }:= − ∈S a bk k Z i skup T S⊂ , { }: 0= ∈ ≥T w S w (skup svih nenegativnih brojeva skupa S).

Skup T nije prazan, jer je 0− ≥a bk čim je ,bk ka

≤ ∈Z .

Kako je skup nenegativnih cijelih brojeva dobro uređen skup*, njegov svaki podskup, pa time i skup T, ima najmanji element, recimo x, pa je tada 0= − ≥x a bk (jer pripada skupu T). Odavde je = +a x bk . Uzmimo da je =x r i =k q . Tada je (*) = +a qk r . Ovim smo dokazali egzistenciju brojeva , ∈q r Z za koje je = +a qk r i 0≥r (jer je 0≥x i =x r ). Dokažimo da je i <r b . Ako bi bilo ≥r b ( 0≥r ) tada je {

0≥

> −r r b , a kako je iz (*) = −r a bq slijedi

( )1 0> − − = − + ≥r a bq b a q b . Dakle, znamo da je r je najmanji element skupa T, a uz pretpostavku ≥r b dobijamo da postoji broj ( ( )1a q b= − + ) iz skupa T (jer je nenegativan a pripada S) koji je manji od r, pa iz ≥r b slijedi da r nije najmanji element skupa T, što je kontradikcija. Zaključujemo da je <r b . Dakle, postoje , ∈q r Z takvi da je 0= + ≤ <a bq r r b . Dokažimo sada jedinstvenost brojeva , ∈q r Z . Pretpostavimo da , ∈q r Z nisu jedinstveni, a to znači da postoje 1 1, ∈q r Z , 1≠q q i 1≠r r takvi da vrijedi:

1 1 1

00

= + ≤ <= + ≤ <

a bq r r ba bq r r b

Oduzimanjem jednakosti dobivamo ( ) ( ) ( )1 1 1 1 10 |= − + − ⇒ − = − ⇒ −b q q r r r r b q q b r r

Kako je 0 ≤ <r b i 10 ≤ <r b odnosno 1r r b− < slijedi da je 1 0− =r r , odnosno 1=r r . Sada imamo ( )1 1 1 1+ = + ∧ = ⇒ =bq r bq r r r q q (što je kontradikcija s pretpostavkom!). Dakle, brojevi , ∈q r Z su jedinstveni. Broj q nazivamo količnikom a broj r ostatkom pri dijeljenju broja a brojem b. Očigledno je, | 0⇔ =b a r . Može se pokazati, ako su , ∈a b Z da postoje jedinstveni brojevi , ∈q r Z takvi da je 0 ≤ <r b . * Za totalno uređen skup ( ),X ≤ kažemo da je dobro uređen skup akko svaki Y X∅ ≠ ⊆ ima minimalni element.

Page 6: Teorija_brojeva_radna_verzija.pdf

6

II PROSTI BROJEVI Definicija 2.1. (prost broj) Prost broj je pozitivan cijeli broj različit od 1 koji nema drugih pozitivnih djelilaca osim brojeva 1 i samog sebe. Definicija 2.2. (složen broj) Pozitivan cijeli broj različit od 1 je složen ako nije prost. Dakle, broj 1 nije ni prost ni složen. U ovom poglavlju ćemo kratko diskutovati distribuciju prostih brojeva i spomenuti neke hipoteze vezane za proste brojeve. U daljem radu ćemo se stalno vraćati prostim brojevima. Kao prvo, dokazati ćemo da postoji beskonačno mnogo prostih brojeva! Daćemo više dokaza ove teorme. Teorema 2.1. Postoji beskonačno mnogo prostih brojeva. Dokaz 1. teoreme 2.1. (potiče od Euklida)

Upotrijebit ćemo indirektni dokaz tj. pretpostavimo da je skup prostih brojeva P konačan, { }2,3,5,7,...,=P p .

Formirajmo broj k na sljedeći način 2 3 5 7 ... 1= ⋅ ⋅ ⋅ ⋅ ⋅ +k p

Očigledno, broj k je neparan broj, što znači da on može biti prost ili složen. Razmotrimo te mogućnosti: 1) Neka je k prost broj. Očigledno, on je veći od svakog broja iz skupa P, što znači da postoji još prostih brojeva koji ne pripadaju skupu P, što je suprotno pretpostavci da je p najveći prost broj. Dakle, k nije prost broj, već je složen broj. 2) Neka je k složen broj. Očigledno on nije djeljiv ni sa jednim brojem iz skupa P, jer pri dijeljenju sa bilo kojim brojem iz skupa P daje ostatak 1. Pošto je k složen broj (po pretpostavci) on se može napisati kao proizvod dva broja od kojih je jedan prost, a to nije ni jedan prost broj iz skupa P, pa znači da van skupa P postoji bar jedan prost broj. Dakle, k nije složen broj (već prost!). Obije ove mogućnosti vode u kontradikciju, odnosno pretpostavka da je P konačan skup nije tačna tj. skup prostih brojeva je beskonačan. Dati ćemo još jedan dokaz ove teoreme ali nam je potrebna sljedeća lema: Lema 2.1. Svaki pozitivan cijeli broj veći od 1 ima prost djelilac. Dokaz

Pretpostavimo da postoji pozitivan cijeli broj veći od 1 koji nema prost djelilac. Posmatrajmo skup S, skup svih prirodnih brojeva većih od 1 koji nemaju prostih djelilaca. Kako smo pretpostavili da postoji pozitivan cijeli broj veći od 1 koji nema prost djelilac to znači da skup S nije prazan, pa skup S ima najmanji element (jer je ( ),≤N dobro uređen skup), recimo n. Iz činjenice da |n n slijedi da n nije prost broj (jer n nema prostih djelilaca).

Page 7: Teorija_brojeva_radna_verzija.pdf

7

Kako n nije prost broj slijedi da je n složen broj tj. da postoje , ∈a b Z takvi da je =n ab i 1< <a n i 1< <b n .

Kako je n najmanji broj koji nema prostog djelioca i <a n slijedi da ∉a S . Međutim, kako skup S obuhvata samo one prirodne brojeve različite od 1 koji nemaju prost djelilac i 1< a je prirodan broj, slijedi da broj a ima prost djelilac, recimo w. Tada, postoji ∈q Z takav da je =a wq , a tada je =n wqb , odnosno n ima prost djelilac. Dakle, skup S je prazan, što je kontradikcija, odnosno, svaki pozitivan cijeli broj veći od 1 ima prost djelilac. Lema 2.2. Neka je ∈a Z takav da je = +a b c , pri čemu su , ∈b c Z . Ako postoji ∈q Z takav da |q a i |q b tada |q c . Dokaz

Neka je ∈a Z i , ∈b c Z takvi da je = +a b c . Neka je |q a . Tada (1) ( )∃ ∈ =s a sqZ . Neka je |q b . Tada (2) ( )∃ ∈ =r b rqZ . Iz = +a b c i (1) i (2) slijedi

{ |∈

⎛ ⎞= + ⇒ = − ⇒⎜ ⎟⎝ ⎠

sq rq c c q s r q cZ

.

Dokaz 2. teoreme 2.1.

Za ∈n N definišimo broj ! 1= +na n . Na osnovu leme 2.1. znamo da na ima bar jedan prost djelilac (jer 1< ∈na N ), recimo nq . Ako bi bilo ≤nq n , tada bi imali:

| !nq n (ovo je očigledno jer ! 1 ... ...= ⋅ ⋅ ⋅ ⋅nn q n i mogućnost da je =nq n ) i kako ( )| ! 1+nq n slijedi na osnovu leme 2.2. da |1nq , a ovo bi značilo da je 1=nq tj. da nq nije ni prost ni složen broj što je u suprotnosti sa činjenicom da on jeste prost broj, a do ovoga nas dovela pretpostavka da je ≤nq n . Dakle >nq n . Odavde zaključujemo, kako je skup prirodnih brojeva beskonačan, a za bilo koji ∈n N postoji nq prost broj takav da je >nq n , da je skup prostih brojeva beskonačan. Pri određivanju prostih faktora datog složenog broja n korisna je sljedeća teorema: Teorema 2.2. Neka je n složen broj. Tada n ima prosti faktor ne veći od n . Dokaz

Pošto je n složen broj možemo pisati =n ab , 1< ≤ <a b n

Neka je a prost broj (to je moguće na osnovu leme 2.1.). Tvrdimo da je ≤a n . Zaista, ako bi bilo tačno >a n tada je (jer je b a> ) > ⋅ =ab n n n , što je apsurdno sa činjenicom da je =n ab . Dakle, ≤a n .

Page 8: Teorija_brojeva_radna_verzija.pdf

8

Koristeći se prethodnom teoremom možemo odrediti sve proste brojeve manje ili jednake od datog broj n. Ta procedura se zove Eratostenovo sito, jer ju je prvi otkrio starogrčki matematičar Eratosten (276 -194). Eratostenovo sito Problem: Dat je prirodan broj n. Odrediti sve proste brojeve manje (ili jednake) od datog broja n. Metoda se sastoji od sljedećeg: Napišu su svi prirodni brojevi počevši od 2 do n (uključujući i njih). Prvo, precrtamo sve brojeve djeljive sa 2, ne precrtavajući 2 (jer je 2 prost). Drugo, precrtamo preostale neprecrtane brojeve djeljive sa 3, ne precrtavajući 3 (jer je 3 prost). Sljedeći neprecrtani broj je 5, on je prost i ostavimo ga, a precrtamo sve neprecrtane brojeve djeljive sa 5. Postupak nastavljamo sa neprecrtanim brojevima sve do broja ⎡ ⎤

⎣ ⎦n (precrtanog ili ne).

Svi ostali brojevi koji nisu precrtani su prosti brojevi! Ovaj postupak, iako prilično jednostavan za primjene, je postao neupotrebljiv u kompjuterskim naukama jer je za ovaj način određivanja prostih brojeva manjih (ili jednakih) od datog broja n potrebno jako mnogo operacija. Danas su razrađene neke druge efikasnije metode. Sada ćemo dokazati da „rupe“ između dva prosta broja mogu biti i po volji velike. ☺ Teorema 2.3. Za dati broj { }\ 1∈n N\ postoji bar n uzastopnih složenih brojeva. Dokaz

Neka je { }\ 1∈n N\ proizvoljan. Tada je svaki od brojeva

( ) ( ) ( ) ( ) ( )2 1 !, 3 1 !, ... , 1 !, 1 1 !n n n n n n+ + + + + + + + +

složen broj, jer je prvi djeljiv sa 2, drugi sa 3, …, n-ti sa ( )1+n i očigledno su jedan drugom direktni sljedbenici (tj. oni su uzastopni brojevi). Iako „rupe“ između prostih brojeva mogu biti po volji velike, s druge strane prosti brojevi mogu biti i veoma blizu. Brojevi 2 i 3 su jedina dva uzastopna prosta broja (jer je 2 jedini prost i paran broj). Međutim, postoje i parovi prostih brojeva koji se razlikuju za 2. Ti brojevi se nazivaju prosti blizanci. Primjeri takvih brojeva su 3, 5; 5, 7; 11, 13; 17, 19; 101, 103; … Jedna od poznatih i nedokazanih hipoteza jeste da postoji beskonačno mnogo prostih blizanaca. Postoji veliki broj nedokazanih teza vezano za proste brojeve. Jedna od njih je sljedeća: - Da li postoji beskonačno mnogo prostih brojeva oblika 2 1+n ? (nije dokazano) Jednu od važnih hipoteza: - U svakom aritmetičkom nizu { } ( )( )1

, , , 1n

a nd a d a d∞

=+ ∈ ∧ =N postoji beskonačno mnogo prostih

brojeva! dokazao je krajem IXX st. njemački matematičar Dirichlet (1805-1959).

Page 9: Teorija_brojeva_radna_verzija.pdf

9

III EUKLIDOV ALGORITAM Definicija 3.1. (zajednički djelilac) Cijeli broj d je zajednički djelilac cijelih brojeva a i b ako |d a i |d b . Skup zajedničkih djelilaca je očigledno neprazan (jer su 1 i -1 djelioci svakog cijelog broja). Ovaj skup je ograničen odozgo pa postoji najveći element. Definicija 3.2. (najveći zajednički djelilac) Najveći zajednički djelilac cijelih brojeva a i b je najveći cijeli broj d koji dijeli i a i b. To označavamo sa ( ), =NZD a b d ili ( ),a b d= . Definicija 3.2. (relativno prosti brojevi) Cijeli brojevi a i b su relativno prosti akko je ( ), 1=NZD a b . Teorema 3.1. Neka su , , ∈a b c Z i neka je ( ), =NZD a b d . Tada je:

a) , 1⎛ ⎞ =⎜ ⎟⎝ ⎠

a bNZDd d

b) ( ) ( ), ,+ =NZD a cb b NZD a b

c) ( )1 1 1| | |∧ ⇒d a d b d d Dokaz

a) Upotrijebićemo indirektni dokaz. Pretpostavimo da postoji cijeli broj 1>e takav da je

| aed

i | bed

.

To znači

( ),∃ ∈ = ∧ =a bk l ke led d

Z a odavde slijedi ( )=a k de i ( )=b l de a odavde zaključujemo da

|de a i |de b . Kako je po pretpostavci teoreme d najveći zajednički djelilac brojeva a i b, odatle slijedi da je

≤de d . Dijeljenjem sa d ( 0>d ) ovu nejednakost dobijamo 1≤e što je kontradikcija s pretpostavkom da je 1>e , pa zaključujemo da je 1=e što je i trebalo

dokazati.

b) Neka je ( ),a b d= i neka je ( ),a cb b e+ = .

Kako |d a i |d b vrijedi |d a cb+ i |d cb . Odavde slijedi d e≤ , jer je ( ),a cb b e+ = .

Kako |e a cb+ i |e b slijedi da |e a . Odavde slijedi da je e d≤ , jer je ( ),a b d= .

Iz d e≤ i e d≤ slijedi da je e d= tj. da je ( ) ( ), ,a b a cb b= + .

c) Neka je ( ),a b d= . Odavde slijedi da |d a i |d b , a odavde zaključujemo

(*) ( ),m n a md b nd∃ ∈ = ∧ =Z , a na osnovu ove teoreme pod a) slijedi ( ), 1m n = . Po pretpostavci je 1 |d a i 1 |d b pa odavde slijedi (**) ( ) 1 1,s t a sd b td∃ ∈ = ∧ =Z

Page 10: Teorija_brojeva_radna_verzija.pdf

10

Iz (*) i (**) slijedi 1 1md sd nd td= ∧ =

Tvrdimo da je 1 |d d . Pretpostavimo suprotno tj. da nije 1 |d d . Tada na osnovu teoreme 1.3. (algoritam dijeljenja) slijedi da postoje jedinstveni brojevi ,q r∈Z takvi da je (***) ( )1 10d qd r r d d= + ≤ < ≤ Kako je 1md sd= , odavde i (***) slijedi

( ) ( )1 |

1 1 1 1 1 1 1| |d r

m qd r sd mqd mr sd d s mq mr d mr d m/

+ = ⇔ + = ⇔ − = ⇒ ⇒ Kako je 1nd td= , odavede i (***) slijedi

( ) ( )1 |

1 1 1 1 1 1 1| |d r

n qd r td nqd nr td d t nq nr d nr d n/

+ = ⇔ + = ⇔ − = ⇒ ⇒

Kako smo dobili da 1 |d m i 1 |d n a važi činjenica ( ), 1m n = tj. m i n su relativno prosti slijedi

1 1d = ili 1 1d = − , a tada u svakom slučaju 1 |d d , a ovo je kontradikcija sa pretpostavkom da

1 |d d/ . Dakle, 1 |d d . Teorema 3.2. (bitno!) Neka je ( ) ( ), 0a bq r b a r b= + < ≤ < . Tada je ( ) ( ), ,a b b r= . Dokaz

Neka je ( ),a b d= i ( ),b r e= .

Tada |d a i |d b , pa slijedi |d bq , te dalje slijedi |d r . Odavde slijedi d e≤ , jer je ( ),b r e= .

Dalje, kako |e b i |e r slijedi |e bq , pa slijedi ( )| |e bq r e a+ ≡ . Odavde slijedi e d≤ , jer je

( ),a b d= .

Kako smo dobili da je d e≤ i e d≤ slijedi e d= tj. ( ) ( ), ,a b b r= . (Euklidov algoritam) Kako naći najveći zajednički djelilac cijelih brojeva a i b (recimo a b> )? Možemo smatrati da su a i b prirodni brojevi jer pitanje djeljivosti ne ovisi od znaka. Primjenjujući algoritam dijeljenja dobijamo, recimo 0a r= i 1b r= (radi zapisa)

( )( )( )

( )

0 1 1 2 2 1

1 2 2 3 3 2

2 3 3 4 4 3

2 1 1 1

1

0

0

0

00

n n n n n n

n n n

r r q r r r

r r q r r r

r r q r r r

r r q r r rr r q− − − −

= + ≤ <

= + ≤ <

= + ≤ <

= + ≤ <

= +

M M M

Navedeni postupak ima konačno mnogo koraka, jer je 0 1 2 3 1 1... 0n n na r r r r r r r− += > > > > > > > = Teorema 3.3. Posljednji ostatak nr koji je različt od nule u prethodnom postupku predstavlja najveći zajednički djelilac brojeva a i b.

Page 11: Teorija_brojeva_radna_verzija.pdf

11

Dokaz Koristeći se teoremom 3.2. dokaz je očigledan. Imamo da je

( ) ( ) ( ) ( ) ( ) ( ) ( )0 1 1 2 2 3 2 1 1, , , , ... , , ,0n n n n n na b r r r r r r r r r r r r− − −= = = = = = = = . Primjer 3.1. Naći ( )252,198 . Koristimo Euklidov algoritam 252 198 1 54198 54 3 3654 36 1 1836 18 2 0

= ⋅ += ⋅ += ⋅ += ⋅ +

Dakle, zadnji ostatak različit od nule je 18, pa na osnovu teoreme 3.3. zaključujemo ( )252,198 18= . Direktna posljedica ove teoreme (posljedica Euklidovog algoritma) je Teorema 3.4 Neka je ( ),a b d= . Tada postoje brojevi ,α β ∈Z takvi da je d a bα β= + . Dokaz

Neka je ( ),a b d= . Koristeći jednakosti iz Euklidovog algoritma dobijamo

2 1 1

1 3 2 2

2 4 3 3

3 5 4 4

3 1 2 2

2 0 1 1 1

n n n n

n n n n

n n n n

n n n n

r r q rr r q rr r q rr r q r

r r r qr r r q a bq

− − −

− − − −

− − − −

− − − −

= −= −

= −= −

= −

= − = −

M M

U prvu jednakost uvrštavamo drugu, pa dobijemo

( ) ( )2 1 1 2 1 3 2 2 2 1 2 3 11n n n n n n n n n n n n n nr r q r r q r q r r q q r q− − − − − − − − − − − − −= − = − − = + − Uz oznake 1 1 21 n nq qα − −= + , 1 1nqβ −= − ova jednakost se može napisati kao

2 1 3 1n n nr r rα β− −= + . U dobijenu jednakost uvrstimo 2 4 3 3n n n nr r q r− − − −= − , pa dobijemo

( ) ( )4 3 3 1 3 1 4 1 3 3 1 3 1 4 1 3 1 3 1n n n n n n n n n n n nr r q r r r q r r r r qα β α α β α β α− − − − − − − − − − −= − + = − + = + − Uz oznake 2 1α α= , 2 1 3 1nqβ β α−= − ova jednakost se može napisati kao

4 2 3 2n n nr r rα β− −= + . Postupak ove zamjene nastavljamo sve do n-2 koraka, kada dobijemo

0 1nr r r a bα β α β= + = + . Kako je na osnovu teoreme 3.3., ( ),nr a b d= = , time je teorema dokazana.

Page 12: Teorija_brojeva_radna_verzija.pdf

12

Primjer 3.2. ( )252,198 18= Iskoristimo primjer 3.1., odakle dobijamo

( ) ( )18 54 36 54 198 54 3 4 54 198 4 252 198 198 4 252 5 198= − = − − ⋅ = ⋅ − = ⋅ − − = ⋅ − ⋅ Dakle, 4α = , 5β = − . Napomena: Reprezentacija broja ( ),d a b= u obliku d a bα β= + data u prethodnom primjeru nije jedinstvena.

Naime, ako je d a bα β= + , onda je ( )b ad k a k b kd d

α β⎛ ⎞ ⎛ ⎞⎜ ⎟ ⎜ ⎟⎝ ⎠ ⎝ ⎠

= + + − ∀ ∈Z .

Teorema 3.5. Ako je ( ),a b d= , tada je d najmanji nenegativan cio broj koji se može napisati u obliku d a bα β= + , ,α β ∈Z . Pojam najvećeg zajedničkog djelioca može se definisati i za više od dva broja. Definicija 3.4. Kažemo da je ( )1 2, ,..., nd a a a= akko je d najveći cijeli broj takav da vrijedi

1 2| | ... | nd a d a d a∧ ∧ ∧ . Teorema 3.6. Vrijedi ( ) ( )( )1 2 1 1 2 1, ,..., , , ,..., ,n n n na a a a a a a a− −= . Na osnovu prethodne teoreme vidimo da se traženje NZD za n brojeva može svesti na traženje NZD za dva broja. Primjer 3.3. ( ) ( )( ) ( )105,198, 252 105, 198, 252 105,18 3= = =

Page 13: Teorija_brojeva_radna_verzija.pdf

13

IV FUNDAMENTALNI TEOREM ARITMETIKE Primjer 4.1. Broj 26460 rastaviti na proste faktore.

Dakle, 2 3 1 226460 2 3 5 7= ⋅ ⋅ ⋅

Teorema 4.1. (FUNDAMENTALNI TEOREM ARITMETIKE – FTA) Svaki pozitivan cijeli broj veći od 1, može se na jedinstven način napisati u obliku proizvoda prostih brojeva

1 21 2 ... k

kp p pαα α⋅ ⋅ ⋅ , 1 2 ... kp p p< < < , ( ), 1, 2,...,i i kα ∈ =N . Za dokaz ove teoreme koristićemo sljedeće dvije leme: Lema 4.1. Neka su ,a b∈N takvi da je ( ), 1a b = i |a bc . Tada |a c . Dokaz

Pošto je ( ), 1a b = slijedi ( ), 1a bα β α β∃ ∈ + =Z . Množeći ovu jednakost sa c, dobijamo (*) ac bc cα β+ = . Kako |a bc slijedi ( )q bc qa∃ ∈ =Z . Uvršatavanjem dobijenog u (*) imamo

( ) |c ac qa c a c q a cα β α β= + ⇔ = + ⇒ . Lema 4.2. Ako 1 2| ... np a a a⋅ ⋅ ⋅ , gdje je p prost broj i 1 2, ,..., na a a ∈N , tada postoji ia takav da | ip a , ( 1, 2,...,i n= ). Dokaz

Dokazat ćemo pomoću matematičke indukcije po n, gdje n označava broj faktora 1 2 ... na a a⋅ ⋅ ⋅ . 1n = (očigledno, jer je tada samo jedan od faktora kojeg p ujedno i dijeli).

Pretpostavimo da je za neko n∈N tačno ( )1 2| ... |n ip a a a p a⋅ ⋅ ⋅ ⇒ za neki 1,2,...,i n= . Tvrdimo da je tačan iskaz ( )1 2 1| ... |n n ip a a a a p a+⋅ ⋅ ⋅ ⋅ ⇒ za neki 1,2,..., , 1i n n= + .

Pretpostavimo da 1| np a +/ , a kako je p prost broj to je ( )1, 1np a + = . Sada, koristeći lemu 4.1., stavimo da je a p= , 1nb a += , 1 2 ... nc a a a= ⋅ ⋅ ⋅ . Tada imamo

.

1 2| ... |pp indukcije

n ip c a a a p a= ⋅ ⋅ ⋅ ⇒ za neki 1,2,...,i n= .

26460 2 13230 2 6615 3 2205 3 735 3 245 5 49 7 7 7 1

Page 14: Teorija_brojeva_radna_verzija.pdf

14

Dakle, dokazali smo da ako 1 2 1| ... n np a a a a +⋅ ⋅ ⋅ ⋅ i 1| np a +/ , tada | ip a za neki 1, 2,...,i n= . No, ako 1| np a + tada je 1i n= + , i time metodom matematičke indukcije zaključujemo da je teorem tačan za svako n∈N . Dokaz FTA

Dokazat ćemo, najprije, egzistenciju broja oblika 1 2

1 2 ... kkp p pαα α⋅ ⋅ ⋅ , 1 2 ... kp p p< < < , ( ), 1, 2,...,i i kα ∈ =N .

Upotrijebit ćemo indirektni dokaz, tj. pretpostavit ćemo da postoji prirodan broj veći od 1 kojeg ne možemo napisati kao proizvod prostih faktora. U tom cilju, neka je n najmanji takav broj. Broj n može biti ili prost ili složen. Ako je n prost, slijedi da je 1n n= , a što je kontradikcija s pretpostavkom, do čega nas dovela pretpostavka da je n prost broj. Dakle, preostaje da n mora biti složen broj. Kako je n složen broj imamo

{ }( )( ), \ 1a b a n b n n ab∃ ∈ < ∧ < =N . Kako je n najmanji broj koji se ne može napisati kao proizvod prostih faktora, i kako je a n< i b n< , slijedi da se brojevi a i b mogu napisati kao proizvodi prostih faktora, a tada je i n proizvod prostih faktora, što predstavlja kontradikciju (sa pretpostavkom da se n ne može napisati kao proizvod prostih faktora). Kako smo iz pretpostavke da postoji prirodan broj veći od 1 kojeg ne možemo napisati kao proizvod prostih faktora, dobili da prirodan broj veći od 1 nije ni prost ni složen (što nije moguće!) slijedi da pretpostavka nije tačna, tj. TVRDNJA TEOREME JE TAČNA! Time je egzistencija dokazana. Dokažimo sada jedinstvenost faktorizacije 1 2

1 2 ... kkp p pαα α⋅ ⋅ ⋅ .

Pretpostavimo da postoji { }\ 1n∈N koji ima dvije različite faktorizacije:

1 2 1 2... , ...s sn p p p p p p= ⋅ ⋅ ⋅ < < <

1 2 1 2... , ...t tn q q q q q q= ⋅ ⋅ ⋅ < < < gdje su ( )1, 2,...,ip i s= i ( )1, 2,...,iq i t= prosti brojevi. Odavde je

1 2 1 2... ...s tp p p q q q⋅ ⋅ ⋅ = ⋅ ⋅ ⋅ Neka je ( )1, 2,...,ip i s= proizvoljan prost faktor. Tada proizvod 1 2 ... tq q q⋅ ⋅ ⋅ mora biti djeljiv brojem ip , jer

( )1 2 1 1 1 2 1 2 1 1 1 2 1 2... ... ... ... ... ... | ...i i i s t i i i s t i tp p p p p p q q q p p p p p p q q q p q q q− + − +⋅ ⋅ ⋅ ⋅ ⋅ ⋅ ⋅ = ⋅ ⋅ ⋅ ⇔ ⋅ ⋅ ⋅ ⋅ ⋅ ⋅ = ⋅ ⋅ ⋅ ⇒ ⋅ ⋅ ⋅. Na osnovu leme 4.2. slijedi da postoji ( )1, 2,...,iq i t= koji je prost i za koji vrijedi |i iq p . Ovo je samo moguće ako je i iq p= . Tako bi za svaki ( )1, 2,...,ip i s= našli iq takav da je i iq p= i obrnuto za svaki ( )1, 2,...,iq i t= postoji ip takav da je i iq p= , odakle ćemo zaključiti da je s t= i i iq p= za svaki 1,2,...,i t s= = . Dakle, ne postoji broj { }\ 1n∈N koji ima dvije različite faktorizacije, odnosno, svaki broj

{ }\ 1n∈N ima jedinstvenu faktorizaciju oblika 1 2

1 2 ... kkp p pαα α⋅ ⋅ ⋅ , 1 2 ... kp p p< < < , ( ), 1, 2,...,i i kα ∈ =N .

Page 15: Teorija_brojeva_radna_verzija.pdf

15

Ako je neki ( ), 1, 2,...,i i kα ∈ =N različit od 1, onda se oblik 1 2

1 2 ... kkn p p pαα α= ⋅ ⋅ ⋅

naziva kanonski oblik prirodnog broja n (kanonska faktorizacija). Faktorizacija broj n u obliku proizvoda prostih faktora je veoma korisna za nalaženje NZD brojeva. Naime, ako je

1 21 2 ... naa a

na p p p= ⋅ ⋅ ⋅ i 1 21 2 ... nbb b

nb p p p= ⋅ ⋅ ⋅ tada je ( ) ( ) ( ) ( )1 1 2 2 min ,min , min ,

1 2, ... n na ba b a bna b p p p= ⋅ ⋅ ⋅

s tim da je ( )1, 2,...,ip i n= faktor akko |ip a i |ip b . Primjer 4.2. Naći NZD za brojeve 240 i 108. Neka je 240a = i 108b = . Izvršimo faktorizaciju brojeva a i b.

4240 2 3 5= ⋅ ⋅

2 3108 2 3= ⋅

Dakle, ( ) 2 1240,108 2 3 12= ⋅ = . (Vidimo da 5 ne dijeli 108, pa i nije u proizvodu!). Definicija 4.1. Najmanji zajednički sadržalac dva prirodna broja a i b u oznaci ( ),NZS a b ili ubuduće [ ],a b je najmanji prirodan broj djeljiv i sa a i sa b. Faktorizacija je pogodna i za nalaženje NZS brojeva. Ako je

1 21 2 ... naa a

na p p p= ⋅ ⋅ ⋅ i 1 21 2 ... nbb b

nb p p p= ⋅ ⋅ ⋅ tada je [ ] ( ) ( ) ( )1 1 2 2 max ,max , max ,

1 2, ... n na ba b a bna b p p p= ⋅ ⋅ ⋅

s tim da je ( )1, 2,...,ip i n= faktor akko |ip a ili |ip b . Kako je ( ) ( )max , min ,x y x y x y+ = + vidimo da je ( ) [ ], ,a b a b a b⋅ = ⋅

240 2 120 2 60 2 30 2 15 3 5 5 1

108 2 54 2 27 3 9 3 3 3 1

Page 16: Teorija_brojeva_radna_verzija.pdf

16

Teorema 4.2. Ako je 1 2

1 2 ... naa ana p p p= ⋅ ⋅ ⋅ faktorizacija broja a, onda je broj pozitivnih djelilaca broja a

(uključujući 1 i a), označavamo sa ( )aτ , ( ) ( ) ( ) ( ) ( )1 2 11 1 ... 1 1

n

n iia a a a aτ

== + ⋅ + ⋅ ⋅ + = Π + .

Dokaz

Dokazat ćemo matematičkom indukcijom. 1n = , tada je 1 1

1aa p a= = pa je broj pozitivnih rješenja ( ) ( )1 1 1 1 2a aτ = + = + = , što je tačno, jer je

tada a prost broj pa ga dijele samo 1 i a. Pretpostavimo da za broj 1 2

1 2 ... naa ana p p p= ⋅ ⋅ ⋅ vrijedi ( ) ( ) ( ) ( )1 21 1 ... 1na a a aτ = + ⋅ + ⋅ ⋅ + .

Tada tvrdimo da za broj ( ) 11 2

1 2 1... n na aa an nb p p p p +

+= ⋅ ⋅ ⋅ ⋅ vrijedi

( ) ( ) ( ) ( ) ( )1 2 11 1 ... 1 1n nb a a a aτ += + ⋅ + ⋅ ⋅ + ⋅ + .

( ) 1 11 21 2 1 1... n n na a aa a

n n nb p p p p a p+ ++ += ⋅ ⋅ ⋅ ⋅ = ⋅

Kako po pretpostavci broj a ima ( ) ( ) ( ) ( )1 21 1 ... 1na a a aτ = + ⋅ + ⋅ ⋅ + , i broj 11

nanp ++ ima 1 1na + +

pozitivnih djelilaca (jer je prost) pa odavde slijedi da je ( ) ( ) ( ) ( ) ( )1 2 11 1 ... 1 1n nb a a a aτ += + ⋅ + ⋅ ⋅ + ⋅ + .

Na osnovu principa matematičke indukcije, zaključujemo da je tvdnja tačna za svako n∈N . Primjer 4.3. Koliko pozitivnih djelilaca ima broj 240?

4240 2 3 5= ⋅ ⋅ pa je 1 2 34, 1, 1a a a= = = , odakle dobijamo da je

( ) ( ) ( ) ( )240 4 1 1 1 1 1 5 2 2 20τ = + ⋅ + ⋅ + = ⋅ ⋅ = (naglašavamo, uključujući 1 i 240).

Page 17: Teorija_brojeva_radna_verzija.pdf

17

V LINEARNE DIOFANTOVE JEDNAČINE Diofant je bio starogrčki matematičar iz III st. n.e. Difantove jednačine se zovu jednačine sa jednom ili više promjenljivih (nepoznatih) čija rješenja pripadaju skupu cijelih brojeva. Najprostiji oblik Diofantove jednačine je

( )0 , ,ax b a b x+ = ∈Z . U zavisnosti od a i b, rješenja mogu biti:

0 ba xa

≠ ⇒ = − (x je rješenje akko x∈Z , i jedinstveno je)

0 0a b x= ∧ ≠ ⇒ ∈∅ , jer ( )0 0x b⋅ = − ≠ , ne postoji takav x.

0 0a b x= ∧ = ⇒ ∈Z , jednačina ima beskonačno mnogo rješenja. Jednačina oblika ( ), ,ax by c a b c+ = ∈Z Ovu jednačinu zovemo linearna Diofantova jednačina sa dvije nepoznate. Analogno ovome, jednačina

( ) ( )1 1 2 2 ... , 1, 2,...,n n ia x a x a x c a c i n+ + + = ∈ =Z je linearna Diofantova jednačina sa n promjenljivih. ∇Teorema 5.1. (bitna za usmeni ispit!) Neka su , ,a b c∈Z i ( ),a b d= . Tada, ako |d c/ tada jednačina ax by c+ = nema rješenja u skupu Z ; ako |d c tada jednačina ax by c+ = ima beskonačno mnogo cjelobrojnih rješenja (rješenja u skupu Z ); ako su 0 0,x y ∈Z neka (konkretna) rješenja jednačine ax by c+ = (takve 0 0,x y ∈Z zovemo partikularna rješenja), tada su sva rješenja te jednačine data sa:

(2) 0

0

bx x tday y td

⎧ = +⎪⎪⎨⎪ = −⎪⎩

gdje je t proizvoljan cijeli broj. Dokaz

Pretpostavimo da jednačina ax by c+ = ima rješenja u skupu Z , recimo x i y. Kako je ( ),a b d= slijedi da |d a i |d b , a odavde slijedi da postoje cijeli brojevi 1d i 2d takvi da je 1a d d= i 2b d d= . Tada imamo:

( )1 2 1 2 |d dx d dy c d d x d y c d c+ = ⇔ + = ⇔ Dakle, ako jednačina ima neko rješenje onda je |d c , odnosno (zakon kontrapozicije) ako |d c/ onda jednačina nema rješenja. Dokažimo, ako |d c tada jednačina ax by c+ = ima beskonačno mnogo cjelobrojnih rješenja. Kako ( ),a b d= i |d c slijedi da |d a i |d b , a odavde slijedi

( )1 2 1 2, ,d d q a d d b d d c qd∃ ∈ = ∧ = ∧ =Z .

Sada, kako je ( ),a b d= slijedi

( ), d a bα β α β∃ ∈ = +Z

Page 18: Teorija_brojeva_radna_verzija.pdf

18

Kako je c qd= i d a bα β= + slijedi

( ) { {0 0

0 0x y

c q a b q a q b a q b q ax byα β α β α β⎛ ⎞ ⎛ ⎞⎜ ⎟ ⎜ ⎟= + = + = + = +⎜ ⎟ ⎜ ⎟⎝ ⎠ ⎝ ⎠

.

Dakle, 0 0c ax by= + , gdje je 0x qα= i 0y qβ= . Ovim smo dokazali postojanje rješenja jednačine ax by c+ = . Da bi dokazali da jednačina ax by c+ = ima beskonačno mnogo rješenja stavimo

0bx x td

= + i 0ay y td

= − , gdje je t proizvoljan cijeli broj.

Tada imamo,

0 0 0 0 0 0b a ab abc ax by a x t b y t ax t by t ax by cd d d d

⎛ ⎞ ⎛ ⎞= + = + + − = + + − = + =⎜ ⎟ ⎜ ⎟⎝ ⎠ ⎝ ⎠

.

Dakle, dokazali smo da je x i y rješenje jednačine ax by c+ = , kojih ima beskonačno mnogo kada t prima vrijednosti iz skupa cijelih brojeva (jer je skup Z beskonačan!). Ostalo je još da pokažemo da su sva rješenja jednačine ax by c+ = oblika (2). Neka je 0 0,x y partikularno rješenje jednačine ax by c+ = . Neka je 1 1,x y proizvoljno rješenje jednačine ax by c+ = . Tada je 0 0ax by c+ = 1 1ax by c+ = Oduzimanjem ove dvije jednačine dobijamo ( ) ( ) ( ) ( ) ( )1 0 1 0 1 0 0 10 : ,a x x b y y a x x b y y d a b− + − = ⇔ − = − = ⇒

{( )

{( ) ( ) ( ) ( )

1 2

1 0 0 1 1 1 0 2 0 1 1 2, 1

d d

a bx x y y d x x d y y d dd d

⇒ − = − ⇒ − = − ∧ =

Odavde slijedi ( )1 0 1|d y y− i ( )2 1 0|d x x− , a odavde dalje slijedi

( ) 1 0 2 1 0 2 1 0bt x x d t x x d t x x td

∃ ∈ − = ⇒ = + ⇔ = +Z

(ili možemo uzeti ( ) 0 1 1t y y d t∃ ∈ − =Z )

Stavljajući dobijeno 1x u jednačinu ( ) ( )1 1 0 2 0 1d x x d y y− = − dobijamo

( ) ( )

( )

( )

1 1 0 2 0 1

1 0 0 2 0 1

0 1

0 1 1 0

:

d x x d y y

bd x t x d y yd

a b b bt y yd d d d

a at y y y y td d

− = − ⇒

⎛ ⎞⇒ + − = − ⇒⎜ ⎟⎝ ⎠

⇒ = − ⇒

⇒ = − ⇒ = −

Ovim smo pokazali da je proizvoljno rješenje 1 1,x y jednačine ax by c+ = oblika (2), tj. sva su rješenja oblika (2). Ova teorema je veoma značajna, pošto nam daje metodu rješavanje jednačine ax by c+ = kao i sva njena cjelobrojna rješenja. To ćemo demonstrirati na sljedećim primjerima.

Page 19: Teorija_brojeva_radna_verzija.pdf

19

Primjer 5.1. Riješiti jednačinu 17 11 20x y+ = . Rješenje Prvo ispitujemo da li jednačina ima rješenje, a na osnovu prethodne teoreme, jednačina će imati rješenje ako ( )17,11 | 20 1| 20⇔ , odakle slijedi da jednačina ima rješenje. U ovom slučaju tražimo partikularno rješenje, i to pomoću Euklidovog algoritma. 17 11 1 611 6 1 56 5 1 15 1 5 0

= ⋅ += ⋅ += ⋅ += ⋅ +

Sada imamo (zbog d a bα β= + ) (posljedica Euklidovog algoritma) ( ) ( )1 6 5 6 11 6 2 6 11 2 17 11 11 2 17 3 11= − = − − = ⋅ − = ⋅ − − = ⋅ − ⋅

Dakle, 2α = i 3β = − .

Sada, 20 201

cqd

= = = , pa je partikularno rješenje:

0 2 20 40x qα= = ⋅ =

0 3 20 60y qβ= = − ⋅ = − Kada smo našli partikularno rješenje napišimo opće rješenje jednačine:

01140 40 111

bx x t t td

= + = + = +

01760 60 171

ay y t t td

= − = − − = − −

Primjer 5.2. Naći partikularno i opće rješenje Diofantove jednačine 1000 123 5x y− = . Rješenje 1000 123 51000 123 8 16123 16 7 1116 11 1 511 5 2 15 1 5 0

x y− == ⋅ += ⋅ += ⋅ += ⋅ += ⋅ +

Vidimo da data jednačina ima rješenje jer ( )1 1000,123= , 1| 5 .

( ) ( )( ) ( )

1 11 5 2 11 16 11 2 3 11 2 16 3 123 16 7 2 16

3 123 23 16 3 123 23 1000 123 8 23 1000 187 123 23 1000 187 123

= − ⋅ = − − ⋅ = ⋅ − ⋅ = − ⋅ − ⋅ =

= ⋅ − ⋅ = ⋅ − − ⋅ = − ⋅ + ⋅ = − ⋅ − ⋅ −

Dakle, 23α = − , 187β = − , 5q = . Partikularna rješenja:

0 23 5 115x = − ⋅ = − , 0 187 5 935y = − ⋅ = − Opća rješenja:

115 123x t= − + , 935 1000y t= − + . Za vježbu riješiti Diofantovu jednačinu 1001 57 10x y+ = .

Page 20: Teorija_brojeva_radna_verzija.pdf

20

Primjer 5.3. (praktičan primjer) Za prevoz neke robe raspolažemo vrećama od 40kg i 60kg. Koliko treba uzetih i jednih i drugih vreća da se preveze 500kg robe? Rješenje Postavimo jednačinu 40 60 500x y+ = , gdje je x broj vreća od 40kg, a y broj vreća od 60kg. Kako su x i y nenegativni cijeli brojevi, to je ova jednačina Diofantova linearna jednačina. Dakle, rješavamo jednačinu 40 60 500 : 202 3 25

x yx y

+ =

+ =

Kako je ( )1 2,3= , 1| 5 slijedi da jednačina ima rješenje. Odredimo jedno rješenje date jednačine „napamet“ (jer su mali koeficijenti) Vidimo da je

0 5x = i 0 5y = . Opća rješenja jednačine glase:

5 3x t= + , 5 2y t= − Uvažimo još uslov da je 0x ≥ i 0y ≥ .

50 5 3 0 3 53

x t t t≥ ⇒ + ≥ ⇒ ≥ − ⇒ ≥ −

50 5 2 0 2 52

y t t t≥ ⇒ − ≥ ⇒ ≤ ⇒ ≤

Dakle, { }5 5 1,0,1,23 2

t t− ≤ ≤ ⇒ ∈ −

Za ove vrijednosti broja t, imamo rješenja: ( ) ( ) ( ) ( ) ( ){ }, 2,7 , 5,5 , 8,3 , 11,1x y ∈ . Primjer 5.4. Naći sve prirodne brojeve koji pri dijeljenju sa 19 daju ostatak 4, a sa 11 ostatak 1. Rješenje Neka je traženi broj z. Iz uslova zadatka dobijamo da je

19 411 1

z xz y= +⎧

⎨ = +⎩

odakle slijedi 19 4 11 1x y+ = + , odnosno 19 11 3x y− = − , a ovo je linearna Diofantova jednačina sa dvije nepoznate. Dovršiti za vježbu. Primjer 5.5 Koje su godine rođene osobe koje su u 1958. godini navršile onoliko godina koliko je zbir cifara godine njihovog rođenja. Rješenje Pretpostavimo da su te osobe rođene 19xy godine, gdje su { }, 0,1, 2,...,9x y∈ . Prema uslovima zadatka mora biti

( )1958 1000 90 10 1 911 2 48

x y x yx y

− + + + = + + +

+ =

Page 21: Teorija_brojeva_radna_verzija.pdf

21

Riješimo sada jednačinu 11 2 48x y+ = Kako je ( )11, 2 1d = , 1| 48 slijedi da jednačina ima rješenje. Jedno partikularno rješenje je

0 2x = , 0 13y = , pa je opće rješenje

( )2 213 11

x tt

y t= +⎧

∈⎨ = −⎩Z .

Međutim, kako je { }, 0,1, 2,...,9x y∈ dobijamo da je

70 2 2 9 2 2 7 1 1 32

t t t t≤ + ≤ ⇔ − ≤ ≤ ⇔ − ≤ ≤ ⇔ − ≤ ≤ i

13 20 13 11 9 13 11 2 1 111 11

t t t t≤ − ≤ ⇔ − ≤ − ≤ − ⇔ ≥ ≥ ⇔ ≥ ≥

odakle slijedi da je 1t = , a tada je 4x = i 2y = , odnosno tražene osobe su rođene 1942. godine. Pokažimo da osoba nije mogla biti rođena neke 18xy godine, jer u tom slučaju dobijamo jednačinu 11 2 149x y+ = , { }, 0,1, 2,...,9x y∈ . Kako je maksimalna vrijednost izraza 11 2x y+ (za 9x = i 9y = ) jednaka 117, to izraz 11 2x y+ ni za koje { }, 0,1,2,...,9x y∈ nije jednak 149, (jer je uvijek manji od 149) odnosno, nema osoba sa navedenim uslovima rođenih 18xy godine.

Page 22: Teorija_brojeva_radna_verzija.pdf

22

Jednačina oblika ( )2 2 2, , , 0ax by cz d a b c d a b c+ + = ∈ ∧ + + ≠Z Rješavanje jednačine (1) ax by cz d+ + = se svodi na rješavanje linearne Diofantove jednačine sa dvije nepoznate ax by c+ = . Prije nego što pređemo na rješavanje jednačine (1) nađimo uvjete uz koje ta jednačina općenito ima rješenja. U tu svrhu, napišimo jednačinu (1) u sljedećem obliku: (2) ax by d cz+ = − pa se problem svodi da se odrede svi cijeli brojevi za koje ta jednačina ima rješenje po x i y. Prema teoremi 5.1. jednačina (2) ima cjelobrojna rješenja samo za one cijele vrijednosti z za koje vrijedi ( ), |a b d cz− , odnosno mora vrijediti

( ),d cz va b−

= ∈Z , odakle slijedi

(3) ( ),cz a b v d+ ⋅ = Jednačina (3) ima cjelobrojna rješenja po z i v (teorema 5.1.) akko vrijedi (4) ( )( ), , |c a b dc odnosno ( ), , |a b c d . Uslov (4) je traženi uslov rješivosti jednačine (1) u skupu Z po x,y,z. Primjer 5.6. Riješiti u skupu Z jednačinu (5) 2 4 5x y z− + = . Rješenje Prvo, ispitajmo uslov rješivosti date jednačine u skupu Z . Pošto je ( )1 2, 4,1= , te 1| 5 slijedi da jednačina ima rješenje. Napišimo jednačinu 2 4 5x y z− + = u obliku 2 4 5x y z− = − Kada će ova jednačina imati cjelobrojna rješenja (po x i y)? Na osnovu teoreme 5.1. slijedi: kako je ( )2 2, 4= , jednačina će imati cjelobrojna rješenja za one cijele z za koje je tačno ( )2 | 5 z− , odnosno ako postoji v∈Z takav da je 2 5 5 2v z z v= − ⇔ = − . Dakle, (6) 5 2z v= − . Uvrstimo (6) u (5), tada dobijamo

( )2 4 5 2 5 2 4 2 : 2 2x y v x y v x y v v− + − = ⇔ − = ⇒ − = ∈Z Očigledno, jedno partikularno rješenje jednačine (6) je

0x v= , 0 0y = ( v∈Z ), pa je opće rješenje jednačine (6)

( )2

,x v t

v ty t= +⎧

∈⎨ =⎩Z .

Iz svega ovoga, opće rješenje jednačine (5) glasi:

( )2

,5 2

x v ty t v tz v

= +⎧⎪ = ∈⎨⎪ = −⎩

Z .

Page 23: Teorija_brojeva_radna_verzija.pdf

23

Zadaci za vježbu 1) Riješiti u skupu Z jednačinu 3 6 5 4x y z− + = . 2) Riješiti u skupu Z jednačinu 7 12 5 100x y z+ − = . 3) Na koliko načina se može novčanica od 100KM usitniti pomoću kovanica od 2, 5 i 10KM?

(Odgovarajuća Diofantova jednačina glasi: 2 5 10 100x y z+ + = )

Page 24: Teorija_brojeva_radna_verzija.pdf

24

VI KONGRUENCIJE Definicija 6.1. Neka je m∈N i ,a b∈Z . Za broj a kažemo da je kongruentan broju b po modulu m i pišemo

( )moda b m≡ akko |m a b− .

Ako |m a b− znači da postoji k∈Z takav da je a b a ba b km k km m m−

− = ⇒ = ⇔ − = .

Ako |m a b−/ onda pišemo ( )moda b m≡/ . Primjer 6.1.

( )23 14 mod 3≡ jer ( )23 14 9 33 3

k−= = = .

A to je jasno i odavde 23 3 7 2= ⋅ + , 14 3 4 2= ⋅ + , pa vidimo da imaju isti ostatak (=2) pri dijeljenju sa 3. Račun s kongruencijama je izuzetno značajan za teoriju brojeva, jer je pomoću kongruencija mnoge teoreme jednostavnije iskazati i dokazati. Dokazaćemo sljedeću tvrdnju: Tvrdnja 6.1. Neka su m∈N i ,a b∈Z . Tada je ( ) ( )moda b m k a km b≡ ⇔ ∃ ∈ = +Z . Dokaz

Neka su m∈N i ,a b∈Z .

(„⇒ “) ( ) ( ).

mod |po def

a b m m a b k a b km a km b≡ ⇒ − ⇒ ∃ ∈ − = ⇒ = +Z

(„⇐“) ( ).

| modpo def

a km b a b km m a b a b m= + ⇒ − = ⇒ − ⇒ ≡ . Tvrdnja 6.2. Neka je m∈N . Kongruencije po modulu m imaju sljedeće osobine: 1) Refleksivnost:

( ) ( )moda a a m∀ ∈ ≡Z 2) Simetričnost: ( ) ( ) ( ), mod moda b a b m b a m∀ ∈ ≡ ⇔ ≡Z 3) Tranzitivnost: ( ) ( ) ( ) ( ), , mod mod moda b c a b m b c m a c m∀ ∈ ≡ ∧ ≡ ⇒ ≡Z . tj. relacija po modulu m je relacija ekvivalencije. Dokaz

Neka je m∈N . 1) Očigledno vrijedi: ( ) ( )| 0 moda m a a a a m∀ ∈ − = ⇒ ≡Z

2) Neka su ,a b∈Z proizvoljni za koje vrijedi

( )moda b m≡ . Odavde slijedi

( ) ( ) ( )mod | | | moda b m m a b m b a m b a b a m≡ ⇔ − ⇔ − − ⇔ − ⇔ ≡ , što je i trebalo dokazati.

Page 25: Teorija_brojeva_radna_verzija.pdf

25

3) Neka su , ,a b c∈Z proizvoljni za koje vrijedi ( )moda b m≡ i ( )modb c m≡ . Odavde slijedi

( ),k l a b km b c lm∃ ∈ − = ∧ − =Z . Sabirajući a b km− = i b c lm− = dobijamo

{ ( )| moda b b c km lm a c m k l m a c a c m∈

⎛ ⎞− + − = + ⇔ − = + ⇒ − ⇒ ≡⎜ ⎟⎝ ⎠Z

, što je i trebalo dokazati.

Ovim je tvrdnja dokazana. Iz ove tvrdnje vidimo da je relacija „≡ “ relacija ekvivalencije koja skup Z dijeli na m disjunktnih skupova koje nazivamo klasama ekvivalencije po modulu m. Klase ekvivalencije su po tvrdnji 6.1. u direktnoj vezi sa ostacima pri dijeljenju nekog broja sa m. Naime, svi brojevi koji pri dijeljenju sa m daju isti ostatak (u smislu algoritma dijeljenja) pripadaju istoj klasi kongruencije po modulu m. Definicija 6.2. Skup cijelih brojeva { }1 2, ,..., mr r r zove se potpuni sistem ostataka po modulu m ako za svaki x∈Z

postoji tačno jedan jr takav da je ( )modjx r m≡ . Drugim riječima, potpun sistem ostataka dobijama tako da iz svake klase ekvivalencije po modulu m uzmemo po tačno jedan član. Kako za dato m vrijedi a bm r= + , 0 r m≤ < tj. 0 1r m≤ ≤ − vidimo da je svaki cijeli broj kongruentan tačno jednom od brojeva iz skupa { }0,1, 2,..., 1m − .

Dakle, skup { }0,1, 2,..., 1m − čini potpun sistem ostataka po modulu m. Sistem ostataka { }0,1, 2,..., 1m − po modulu m je tzv. sistem najmanjih nenegativnih ostataka. Postoje i drugi sistemi ostataka. Jedan od njih je tzv. sistem ostataka najmanjih po modulu, koji za m∈N neparan broj čine sistem

1 3 3 1, ,..., 1,0,1,..., ,2 2 2 2

m m m m− − − −− − −

a za m∈N paran broj postoje dva takva sistema: 2 2, ,..., 1,0,1,...,

2 2 2m m m− −

− − − i

2 2,..., 1,0,1,..., ,2 2 2

m m m− −− − .

Sada ćemo dokazati nekoliko teorema koje kazuju zašto je račun kongruencija veoma koristan.

Page 26: Teorija_brojeva_radna_verzija.pdf

26

Teorema 6.1. Neka su , ,a b c∈Z , m∈N i neka je ( )moda b m≡ . Tada vrijedi:

a) ( )moda c b c m+ ≡ +

b) ( )moda c b c m− ≡ −

c) ( )modac bc m≡ . Dokaz

a) ( ) ( ) ( ) ( ) ( ) ( )mod | | | moda b m m a b m a c c b m a c c b a c b c m≡ ⇔ − ⇒ + − − ⇒ + − + ⇔ + ≡ +⎡ ⎤⎣ ⎦ b

b) ( ) ( ) ( ) ( ) ( ) ( )mod | | | moda b m m a b m a c b c m a c b c a c b c m≡ ⇔ − ⇒ − − + ⇒ − − − ⇔ − ≡ −⎡ ⎤⎣ ⎦ c

c) ( ) ( ) ( ) ( ) ( )mod | | | moda b m m a b m c a b m ac bc ac bc m≡ ⇔ − ⇒ − ⇒ − ⇔ ≡ . Iz prethodne teoreme vidimo da kongruencije možemo množiti cijelim brojem. Postavlja se pitanje, da li možemo dijeliti cijelim brojem?. Odgovor je negativan, jer npr.

( )20 2 mod 6≡ ali (dijeljenjem sa 2) ( )10 1 mod 6≡ . Međutim, nešto je ipak moguće reći i u ovom slučaju, kao što je sljedeća teorema. Teorema 6.2. Neka su , ,a b c∈Z , m∈N i neka je ( ),d c m= . Tada vrijedi:

( )mod mod mac bc m a bd

⎛ ⎞≡ ⇒ ≡ ⎜ ⎟⎝ ⎠

Dokaz

Neka su , ,a b c∈Z , m∈N i neka je ( ),d c m= .

Neka je ( )modac bc m≡ . Dokažimo da je mod ma bd

⎛ ⎞≡ ⎜ ⎟⎝ ⎠

.

Kako je ( )modac bc m≡ slijedi ( )|m ac bc− , a odavdje dalje slijedi ( )|m c a b− , a ovo znači da postoji k ∈Z takav da je ( )c a b km− =

Dijeljenjem sa d dobijenu jednakost, dobijamo:

( )c ma b kd d

− = , te kako je, na osnovu teoreme 3.1. pod a), , 1c md d

⎛ ⎞ =⎜ ⎟⎝ ⎠

, dobijamo:

( )m a bd

− , a ovo znači da je mod ma bd

⎛ ⎞≡ ⎜ ⎟⎝ ⎠

, što je i trebalo dokazati.

Specijalno, ako je ( ), 1c m = , tj. ako su c i m relativno prosti brojevi tada tvrđenje teoreme vrijedi i

glasi ( ) ( )mod modac bc m a b m≡ ⇒ ≡ .

Page 27: Teorija_brojeva_radna_verzija.pdf

27

Primjer 6.2. Izvršiti „skraćivanje“ tj. primjeniti prethodnu teoremu na kongruenciju ( )120 45 mod15≡ . Rješenje Podijelimo prvo sa 5. Kako je ( ) ( ), 5,15 5c m = = , po prethodnoj teoremi dobijamo:

( )120 45 15mod 24 9 mod35 5 5

⎛ ⎞≡ = ≡⎜ ⎟⎝ ⎠

.

Idući dalje, tj. dijeljenjem sa 3 dobijamo ( )8 3 mod1≡ .

Dakle, ( )120 45 mod15≡ je isto što i ( )8 3 mod1≡ . Teorema 6.3. Neka su , , ,a b c d ∈Z , m∈N i neka je ( )moda b m≡ i ( )modc d m≡ . Tada vrijedi:

a) ( )moda c b d m+ ≡ +

b) ( )moda c b d m− ≡ −

c) ( )modac bd m≡ Dokaz

Neka su , , ,a b c d ∈Z , m∈N i neka je ( )moda b m≡ i ( )modc d m≡ .

a) Iz ( )moda b m≡ i ( )modc d m≡ slijedi ( )|m a b− i ( )|m c d− , a ovo znači da postoje ,k l∈Z takvi da je

a b kmc d lm− =− =

Međusobno sabirajući dobijene jednakosti dobijamo ( ) ( ) ( )

( ) ( ) ( )| mod

a b c d km lm a c b d m k l

m a c b d a c b d m

− + − = + ⇒ + − + = + ⇒

⇒ + − + ⇒ + ≡ +

b) Iz ( )moda b m≡ i ( )modc d m≡ slijedi ( )|m a b− i ( )|m c d− , a ovo znači da postoje

,k l∈Z takvi da je a b kmc d lm− =− =

Oduzimajući, recimo od prve drugu od dobijenih jednakost dobijamo ( ) ( ) ( ) ( )( ) ( ) ( )| mod

a b c d km lm a b c d km lm a c b d m k l

m a c b d a c b d m

− − − = − ⇒ − − + = − ⇒ − − − = − ⇒

⇒ − − − ⇒ − ≡ −

c) Iz ( )moda b m≡ i ( )modc d m≡ slijedi ( )|m a b− i ( )|m c d− , a ovo znači da postoje

,k l∈Z takvi da je a b km− = i c d lm− = Odavde ćemo izraziti a i c, pa izvršiti množenje a sa c. a km bc lm d= += +

( )( )ac km b lm d= + + . Oduzimajući bd, dobijamo:

( )( ) ( )2ac bd km b lm d bd klm dkm blm bd bd m klm dk bl− = + + − = + + + − = + + ⇒

( ) ( )| modm ac bd ac bd m⇒ − ⇔ ≡ .

Page 28: Teorija_brojeva_radna_verzija.pdf

28

Teorema 6.4. Ako je { }1 2, ,..., mr r r potpun sistem ostataka po modulu m, a∈N i ( ), 1a m = , tada je

{ }1 2, ,..., mar b ar b ar b+ + + također potpun sistem ostataka po modulu m za proizvoljan cijeli broj b. Dokaz

Na osnovu definicije 6.2. jasno je da ovdje treba dokazati da nikoja dva od elemenata (brojeva) skupa { }1 2, ,..., mar b ar b ar b+ + + nisu kongruentna po modulu m. Ako bi bili kongruentni, tj. ako bi bilo

( )modi jar b ar b m+ ≡ + za neke { }, 1, 2,...,i j m∈ i i j≠ , tada na osnovu teoreme 6.1. slijedi

( )modi jar ar m≡ , a kako je ( ), 1a m = dalje slijedi (teorema 6.2.) ( )modi jr r m≡ .

Međutim, ( )modi jr r m≡ povlači da skup { }1 2, ,..., mr r r nije potpun sistem ostataka po modulu m što je nemoguće. Dakle, nikoja dva od elemenata skupa { }1 2, ,..., mar b ar b ar b+ + + nisu kongruenta, pa stoga (definicija 6.2.) oni čine potpun sistem ostataka po modulu m, što je i trebalo dokazati. Teorema 6.5. (veoma korisna) Ako su ,a b∈Z ; ,k m∈N i ( )moda b m≡ . Tada je ( )modk ka b m≡ Dokaz

Iz ( )moda b m≡ slijedi ( )|m a b− , te kako je

( )( ) ( )1 2 2 1...n n n n n na b a b a a b ab b n− − − −− = − + + + + ∀ ∈N

slijedi ( ) ( )| modn n n nm a b a b m− ⇒ ≡ , a kako ovo vrijedi za svako n∈N , teorema je dokazana! Teorema 6.6. (Mirnesova teorema – o djeljivosti) Neka su 1 2,m m ∈N prosti i x∈Z . Ako je 1 |m x i 2 |m x onda je 1 2 |m m x⋅ . Dokaz

Kako je 1 |m x i 2 |m x to znači da postoje cijeli brojevi 1 2,k k takvi da vrijedi

1 1k m x= i 2 2k m x= , odnosno 1 1 2 2k m k m x= = . Iz 1 |m x i 2 2k m x= slijedi 1 2 2|m k m . Kako su 1 2,m m prosti (pa time i relativno prosti), na osnovu leme 4.2. slijedi da 1 2|m k , a ovo znači da postoji cijeli broj 1l takav da je 2 1 1k l m= . Iz 2 2k m x= i 2 1 1k l m= slijedi 1 1 2l m m x= , a ovo znači da je 1 2 |m m x što je i trebalo dokazati. Lahko se dokazuje (matematičkom indukcijom), da ovo tvrđenje vrijedi i za n prostih brojeva! Teorema 6.7. Neka su ,a b∈Z i 1 2, ,..., km m m ∈N takvi da vrijedi:

( )( )

( )

1

2

mod

mod

mod k

a b m

a b m

a b m

M

Tada je [ ]( )1 2mod , ,..., ka b m m m≡ .

Page 29: Teorija_brojeva_radna_verzija.pdf

29

Dokaz Najprije ćemo dokazati sljedeću lemu:

Lema 6.1. Neka je x∈Z i 1 2,m m ∈N . Ako 1 |m x i 2 |m x onda [ ]1 2, |m m x . Dokaz leme Ako je bar jedan od 1 2,m m jednak 1, tada je očigledna tačnost tvrdnje. Ako su 1 2,m m oba različiti od 1, imamo: kako 1 |m x i 2 |m x slijedi da postoje brojevi 1 2,k k ∈Z takvi da je

1 1

2 2

x k mx k m==

Na osnovu FTA za brojeve 1 2,m m imamo: 1 2

1 1 2 ... kkm p p pαα α= ⋅ ⋅ ⋅ i 1 2

2 1 2 ... kkm p p pββ β= ⋅ ⋅ ⋅ .

Tada imamo: 1 2 1 2

1 1 2 2 1 1 2 2 1 2... ...k kk kx k m k m k p p p k p p pα βα α β β= = = ⋅ ⋅ ⋅ ⋅ = ⋅ ⋅ ⋅ ⋅

Odavde slijedi ( )max ,| | |i jji

i j ip x p x p xα ββα ∧ ⇒ za svako { }, 1, 2,...,i j k∈ i i j= . Kako su 1 2, ,..., kp p p relativno prosti slijedi

( ) ( ) ( )1 1 2 2 max ,max , max ,1 2 ... |k k

kp p p xα βα β α β⋅ ⋅ ⋅ a to znači da je [ ]1 2, |m m x , što je trebalo dokazati u lemi. Dokaz teoreme Kako je [ ] [ ]1 2 1 1 2 1, ,..., , , ,..., ,k k k km m m m m m m m− −⎡ ⎤= ⎣ ⎦ , matematičkom indukcijom sada lahko dokazujemo da za x∈Z vrijedi:

[ ]1 2 1 2| | ... | , ,..., |k km x m x m x m m m x∧ ∧ ∧ ⇒ . Dokaz teoreme je sada trivijalan. Dovoljno je staviti x a b= − , a tada imamo:

[ ] [ ]( )1 2 1 2 1 2| | ... | , ,..., | mod , ,...,k k km a b m a b m a b m m m a b a b m m m− ∧ − ∧ ∧ − ⇒ − ⇒ ≡ čime je teorema dokaza. Ova teorema ima veoma korisnu posljedicu: Posljedica 6.1. Neka su 1 2, ,..., km m m ∈N u parovima relativno prosti i neka za ,a b∈Z vrijedi:

( )( )

( )

1

2

mod

mod

mod k

a b m

a b m

a b m

M

Tada je ( )1 2mod ... ka b m m m≡ ⋅ ⋅ ⋅ .

Page 30: Teorija_brojeva_radna_verzija.pdf

30

Primjer 6.3. Fermaov broj 22 1

n

nF = + je prost za 0,1,2,3,4n = , to je i sam on pokazao. Međutim, pogrešno je tvrdio da su i svi ostali brojevi prosti. Veliki švicarski matematičar Leonhard Euler (1707-1783) je nakon više od 80. godina pokazao da je broj

52 325 2 1 2 1F = + = + složen i da ima djelilac 641, tj.

5 4294967297 641 6700417F = = ⋅ Dokažimo sada pomoću kongruencija da je 5641| F . Rješenje (profesor dokazao!)

( )( )

10

6

2 1024 383 mod 641

2 64 64 mod 641

= ≡

= ≡

Na osnovu teoreme 6.3. pod c) (o množenju kongruencija) slijedi:

( ) ( )10 6 162 2 383 64 mod 641 2 24512 mod 641⋅ ≡ ⋅ ⇒ ≡ (1)

( )24512 154 mod 641≡ (2) Iz (1) i (2) slijedi (zbog osobine relacije ekvivalencije)

( )162 154 mod 641≡ , a odavde je na osnovu teoreme 6.5.

( )32 22 154 mod 641≡ (3)

( )2154 23716 640 mod 641= ≡ (4) Iz (3) i (4) slijedi

( )322 640 mod 641≡ (5)

Zbog ( )640 1 mod 641≡ − i (5) slijedi

( )322 1 mod 641≡ − a ovo znači da je 325641| 2 1 F+ = što je i trebalo pokazati!

Primjer 6.4. Naći ostatak pri dijeljenju broja 1566 sa 7. Rješenje

( ) ( )15 1566 7 9 3 66 3 7 9 7 | 66 3 66 3 mod 7 66 3 mod 7= ⋅ + ⇒ − = ⋅ ⇒ − ⇒ ≡ ⇒ ≡ .

Brojevi 1566 i 153 imaju isti ostatak pri dijeljenju sa 7. Sada tražimo ostatak pri dijeljenju broja 153 sa 7. Kako je ( )515 3 53 3 27= = imamo:

( ) ( )5 527 7 3 6 27 6 mod 7 27 6 mod 7= ⋅ + ⇒ ≡ ⇒ ≡

Vidimo da brojevi 527 i 56 imaju isti ostatak pri dijeljenju sa 7, pa tražimo ostatak pri dijeljenju 56 sa 7. Kako je ( ) ( ) ( ) ( )55 56 1 mod 7 6 1 mod 7 6 1 mod 7≡ − ⇒ ≡ − ⇒ ≡ − . Dakle, traženi ostatak je jednak ostatku pri dijeljenju 1− sa 7. Pošto je ( )1 7 1 6− = ⋅ − + zaključujemo da je ostatak pri dijeljenju 1566 sa 7 jednak 6.

Page 31: Teorija_brojeva_radna_verzija.pdf

31

Primjer 6.5. Naći ostatak pri dijeljenju broja 80 8030 7N = + sa 11. Rješenje Napišimo broj N na sljedeći način:

( ) ( )20 2080 80 4 4 20 2030 7 3 7 81 2401N = + = + = +

( ) ( )20 2081 4 mod11 81 4 mod11≡ ⇒ ≡

( ) ( )20 202401 3 mod11 2401 3 mod11≡ ⇒ ≡ Sabiranjem ovih kongruencija dobijamo:

( )( )20 20 20 2081 2401 4 3 mod11+ ≡ +

Dakle, broj N pri dijeljenju sa 11 ima isti ostatak kao i pri dijeljenju 20 204 3+ sa 11.

( ) ( )4 420 20 5 5 4 44 3 4 3 1024 243+ = + = +

Kako je ( )1024 1 mod11≡ i ( )243 1 mod11≡ to je

( )( ) ( )4 4 4 41024 243 1 1 mod11 2 mod11+ ≡ + = . Dakle, traženi ostatak je 2. Primjer 6.6 Naći ostatak pri dijeljenju broja 16 25 153 2 5N = − ⋅ sa 3. Za vježbu! Pri rješavanju zadataka u kojima treba odrediti zadnju cifru nekog stepenovanog cijelog broja, često se koristi pravilo supstitucije, koje glasi ( ) ( ) ( )mod mod moda bc m b d m a dc m≡ ∧ ≡ ⇒ ≡ Primjer 6.7. Odrediti zadnje dvije cifre broja 1002N = . Rješenje Zadnje dvije (tri) cifre datog broja su zapravo ostatak koji se dobije dijeljenjem tog broja sa 100 (1000). Zadatak se svodi na određivanje tog ostatka.

( )10100 10 102 2 1024N = = =

Kako je ( ) ( )10 101024 24 mod100 1024 24 mod100≡ ⇒ ≡ , traženi ostatak jednak je ostatku koji se

dobije pri dijeljenju 1024 sa 100.

( )510 2 524 24 576= =

Kako je ( )576 24 mod100≡ − slijedi ( ) ( )55576 24 mod100≡ − . Međutim, to možemo napisati i ovako:

{ ( )( )4

5 2

24

576 576 24 mod100≡ ⋅ − (*)

Kako je ( ) ( )22576 24 mod100≡ − , pa uvrštavanjem u (*) a na osnovu pravila supstitucije dobijamo

( ) ( ) ( )( )35576 24 mod100 576 24 mod100≡ − = ⋅ − . Kako je još ( )576 24 mod100≡ − primjenom

supstitucije dobijamo ( ) ( )25576 24 mod100= − .

Odavde slijedi da broj N tj. 5576 ima isti ostatak pri dijeljenju sa 100 kao i ( )224 576− = pri dijeljenju sa 100. Taj ostatak je, očigledno 76.

Page 32: Teorija_brojeva_radna_verzija.pdf

32

VII LINEARNE KONGRUENCIJE Definicija 7.1. Kongruenciju oblika

( )modax b m≡ (1) gdje je x nepoznat cijeli broj nazivamo linearnom kongruencijom u jednoj varijabli. Primijetimo najprije da ako je 0x rješenje kongruencije (1), te ako je ( )1 0 modx x m≡ tada je

( ) ( )1 0 mod modax ax m b m≡ = , odnosno i 1x je rješenje kongruencije (1). Dakle, ako je jedan član klase kongruencije po modulu m rješenje jednačine (1) tada je to i svaki član te klase. Zbog toga je od interesa posmatrati koje klase su rješenja kongruencije (1), tj. koliko takvih različitih klasa ima. Teorema 7.1. Neka su ,a b∈Z , m∈N i ( ),d a m= .

Ako |d b/ tada ( )modax b m≡ nema rješenja.

Ako |d b tada ( )modax b m≡ ima tačno d rješenja nekongruentnih po modulu m. Dokaz

Iz ( )modax b m≡ slijedi |m ax b− a ovo znači da posoji y∈Z takav da je my ax b ax my b= − ⇒ − = . Dakle, x je rješenje jednačine (1) ako postoji y∈Z tako da je ax my b− = (2) Ovo je linearna Diofantova jednačina o čijem rješenju je govorila teorema 5.1. . Jednačina (2) ima rješenje akko |d b . Neka |d b , i neka su ( )0 0,x y neka partikularna rješenja jednačine (2). Tada ( )0 0,x y mora zadovoljavati (2), pa mora biti:

0 0ax my b− = (3) Ako od jednačine (2) oduzmemo jednačinu (3), dobit ćemo: ( ) ( )0 0 0a x x m y y− − − =

odnosno: ( )0

0

a x xy y

m−

− =

Kako je (po pretpostavci teoreme) ( ),d a m= , podijelit ćemo i brojnik i nazivnik desne strane prethodne jednakosti sa d, tada ćemo dobiti:

( )0

0

a x xdy y m

d

−− =

Page 33: Teorija_brojeva_radna_verzija.pdf

33

Kako je u prethodnoj jednakosti 0y y− uvijek cijeli broj, to i desna strana mora biti takva, a to će biti akko je:

kako je , 1a md d

⎛ ⎞ =⎜ ⎟⎝ ⎠

, to mora biti:

0

0 0 ,x x m mt x x t x x t tm d dd

−= ⇒ − = ⇒ = + ∈Z

Rješenja ima beskonačno, međutim dokažimo da ih ima d nekongruentnih. Ako su dva rješenja kongruentna po modulu m, tada je:

( ) ( )0 1 0 2 1 2mod modm m m mx t x t m t t md d d d

+ ≡ + ⇒ ≡

Kako je ( ),d a m= slijedi , m mmd d

⎛ ⎞ =⎜ ⎟⎝ ⎠

, a na osnovu teoreme 6.2. slijedi:

( )1 2 1 2mod modmt t t t dmd

⎛ ⎞⎜ ⎟/≡ ⇒ ≡⎜ ⎟/⎜ ⎟⎝ ⎠

.

Dakle, skup nekongruentnih rješenja dobijamo da je za { }0,1, 2,..., 1t d∈ − , tj. sva nekongruentna rješenja su:

0mx x td

= + , { }0,1, 2,..., 1t d∈ − i ima ih tačno d.

Primjer 7.1. Naći nekogruentna rješenja jednačine ( )9 12 mod15x ≡ . Rješenje ( ) ( ), 9,15 3a m = = Pošto 3 |12 slijedi da postoji tačno tri nekogruentna rješenja. Nađimo ih! Posmatrat ćemo odgovarajuću Diofantovu jednačinu 9 15 12 3 5 4x y x y− = ⇔ − = Očigledno, jedno partikularno rješenje je 0 3x = , 0 1y = . Sada vidimo da je skup nekongruentnih rješenja po modulo 15:

1 3x =

2 8x =

3 13x = Ili, drugačije zapisano, rješenje je 3 5x t= + , 0,1, 2t = . Ukoliko posmatramo specijalne kongruencije ( 1b = ) oblika:

( )1 modax m=

Vidimo da ova kongruencija ima rješenje akko ( ), 1a m d= = . U tom slučaju rješenje je jedinstveno po modulu m.

Page 34: Teorija_brojeva_radna_verzija.pdf

34

Definicija 7.2. (inverz) Neka je ( ), 1a m = . Rješenje x kongruencije ( )1 modax m≡ zovemo inverzom broja a modulo m. Jasno je, da inverz broja a modulo m nije jedinstven, tj. ako je a inverz broja a modulo m, tada je i svaki 'a koji zadovoljova kongruenciju ( )' moda a m≡ inverz broja a modulo m. Dalje, jasno je da svaki broj a iz definicije ima svoj inverz po modulu m, jer kongruencija

( )1 modax m≡ uvijek ima rješenje (po teoremi 7.1.) jer ( ), 1|1a m = . Ukoliko znamo inverz broja a modulo m, možemo ga koristiti za rješavanje linearne kongruencije

( )modax b m≡ .

Neka je a inverz broja a modulo m, tj. neka je ( )1 modaa m= . Tada je

( ) ( ) ( ) ( )mod mod modax b m aa x ba m x ba m≡ ⇒ ≡ ⇒ ≡ . Posebno važan slučaj inverza broja a modulo m jeste kada je m prost broj! U ovom slučaju vrijedi sljedeća teorema. Teorema 7.2. Neka je p prost broj. Prirodan broj a je svoj inverz modulo p akko

( )1 moda p≡ ili ( )1 moda p≡ − . Dokaz

Kako je p prost broj, to je ( ), 1a p = za bilo koji a∈N , a ovo je potrebno za uslov definicije 7.2.. „⇒ “ Neka je a svoj inverz, tj. neka je a a= . Tada je (po definiciji inverza)

( )1 modaa p≡ , a kako je a a= , to je ( )2 1 moda p≡ .

Odavde slijedi, kako je ( )2 1 moda p≡ , a to znači da 2| 1p a − odnosno ( )( )| 1 1p a a− + , a ovo

zadnje je ispunjeno akko ( )| 1p a − ili ( )| 1p a + , odnosno (preko kongruencija)

( )1 moda p≡ ili ( )1 moda p≡ − , što je trebalo pokazati.

„⇐“ Neka je (redom) ( )1 moda p≡ ili ( )1 moda p≡ − . Tada na osnovu teoreme 6.5. slijedi da je

( ) ( )22 1 moda p≡ − ili ( )2 21 moda p≡ , odnosno da je ( )1 moda a p⋅ ≡ , a odavde (na osnovu definicije) slijedi da je a a= , što je trebalo dokazati.

Page 35: Teorija_brojeva_radna_verzija.pdf

35

VIII KINESKA TEOREMA O OSTACIMA (KTO) U ovom poglavlju ćemo posmatrati sisteme linearnih kongruencija čiji su moduli različiti. Prvo ćemo dokazati KTO. Teorema 8.1.(KTO) (nezaobilazna na usmenom ispitu) Neka su 1 2, ,..., km m m u parovima relativno prosti prirodni brojevi, te neka su 1 2, ,..., ka a a cijeli brojevi. Tada sistem kongruencija

( )( )

( )

( )

1 1

2 2

mod

mod1

modk k

x a m

x a m

x a m

≡ ⎫⎪

≡ ⎪⎬⎪⎪≡ ⎭

M

ima rješenje. Ako je 0x neko rješenje sistema (1) onda je svako rješenja sistema (1) dato sa

( )0 1 2 ... kx x m m m≡ ⋅ ⋅ ⋅ . Dokaz

Prvo konstruišimo rješenje sistema (1). Stavimo da je 1 2 ... kM m m m= ⋅ ⋅ ⋅ , zatim

{ }( )1 2 1 1... ...1, 2,..,j j j k

jj j

m m m m m mMM j km m

− +⋅ ⋅ ⋅ ⋅ ⋅ ⋅ ⋅= = ∈

Kako su 1 2, ,..., km m m u parovima relativno prosti, to je ( ), 1j jM m = .

Tada postoji jy ∈Z takav da je

( )1 modj j jM y m≡ .

Posmatrajmo sada sumu (ova suma uvijek postoji, jer uvijek postoje jy ∈Z )

0 1 1 1 2 2 2 ... k k kx a M y a M y a M y= + + + Pokažimo da je 0x rješenje sistema (1), tj da je ( ) ( )0 mod 1,2,...,j jx a m j k≡ ∀ = . Dakle, treba dokazati da je

( ) ( )1 1 1 2 2 2 ... mod 1,2,...,k k k j ja M y a M y a M y a m j k+ + + ≡ ∀ =

Jasno je da ( )|j l l lm a M y za l j≠ , tj.

(*) ( ) ( )0 modl l l ja M y m za l j≡ ≠

treba još pokazati da |j j j jm a M y .

Iz ( )1 modj j jM y m≡ , množeći kongruenciju sa ja dobijamo

(**) ( )modj j j j ja M y a m≡ . Dakle, sada sabirajući (*) za sve l j≠ dobijamo da je (***) ( )1 1 1 1 1 1 1 1 1... ... 0 modj j j j j j k k k ja M y a M y a M y a M y m− − − + + ++ + + + ≡ . Sabirajući (***) i (**) dobijamo

( ) ( )0 mod 1,2,...,j jx a m j k≡ ∀ = , što znači da je 0x rješenje sistema (1).

Page 36: Teorija_brojeva_radna_verzija.pdf

36

Ostaje još da pokažemo da su svaka dva rješenja sistema (1) kongruentna po modulu M. Neka su 1 2,x x dva rješenja sistema (1). Tada je

( ) ( )1 2 mod 1,2,...,j jx x a m j k≡ ≡ ∀ = . Odavde slijedi, (na osnovu same definicije kongruencije)

( )2 1| 1, 2,...,jm x x j k− ∀ = . Kako su 1 2, ,..., km m m u parovima relativno prosti, tada na osnovu posljedice 6.1. slijedi da

1 2 2 1... |km m m x x⋅ ⋅ ⋅ − , a ovo znači da je

( )1 2 modx x M≡ . Dakle, ako su 1 2,x x bilo koja dva rješenja sistema (1) tada su ona kongruentna po modulu M. Ovim je teorema dokaza. Primjer 8.1. (važan primjer za razumijevanje KTO) Odrediti najmanji prirodan broj koji pri dijeljenju sa 6 daje ostatak 4, pri dijeljenju sa 7 daje ostatak 5, a pri dijeljenju sa 11 daje ostatak 6. Rješenje Dakle, treba riješiti sistem linearnih kongruencija

( )( )( )

4 mod 6

5 mod 7

6 mod11

x

x

x

Imamo da je 6 7 11 462M = ⋅ ⋅ = .

11

462 776

MMm

= = =

22

462 667

MMm

= = =

33

462 4211

MMm

= = =

Sada treba riješiti sljedeće kongruencije: ( )177 1 mod 6y ≡

( )266 1 mod 7y ≡

( )342 1 mod11y ≡ Njihova rješenja nalazimo koristeći obrat Euklidovog algoritma, ustvari, tražimo rješenja linearnih Diofantovih jednačina:

1

'177 6 1y y− =

2

'266 7 1y y− =

3

'342 11 1y y− =

Dakle, (obrat Euklidovog algoritma)

( ) ( )77 6 12 5

1 6 5 6 77 6 12 13 6 17 16 5 1 1

= ⋅ + ⎫⎪ = − = − − ⋅ = ⋅ + ⋅ −⎬= ⋅ + ⎪⎭

, 1 1y = − .

Page 37: Teorija_brojeva_radna_verzija.pdf

37

( )66 7 9 3

1 7 3 2 7 66 7 9 2 2 66 19 77 3 2 1

= ⋅ + ⎫⎪ = − ⋅ = − − ⋅ ⋅ = − ⋅ + ⋅⎬= ⋅ + ⎪⎭

, 2 2y = − .

( ) ( )( )42 11 3 911 1 9 2 1 9 4 2 9 4 11 9 9 4 11 42 11 3 ... 5 42 19 11

9 4 2 1

⎫= ⋅ +⎪

= ⋅ + = − ⋅ = − ⋅ − = − ⋅ − − ⋅ = = ⋅ − ⋅⎬⎪= ⋅ + ⎭

, 3 5y = .

Traženo rješenje je

( ) ( )4 77 1 5 66 2 6 42 5 308 660 1260 292x = ⋅ ⋅ − + ⋅ ⋅ − + ⋅ ⋅ = − − + = Dakle, 292x = je rješenje datog sistema.

Page 38: Teorija_brojeva_radna_verzija.pdf

38

IX SISTEMI LINEARNIH KONGRUENCIJA U prethodnom poglavlju razmatrali smo sisteme linernih kongruencija sa jednom nepoznatom, ali različitih i u parovima relativno prostih modula. Možemo posmatrati sisteme linearnih kongruencija po istim modulu, sa onoliko jednačina koliko je nepoznatih i rješavati ih na način analogan klasičnom sistemu linearnih jednačina. Sada razmatramo sisteme sa dvije nepoznate. Teorema 9.1. Neka su , , , , ,a b c d e f ∈Z i m∈N , tako da je ( ), 1mΔ = , gdje je ad bcΔ = − . Tada sistem kongruencija

( )( )mod

mod

ax by e m

cx dy f m

+ ≡

+ ≡

ima jedinstveno rješenje po modulu m dato sa ( )( )( )( )

mod

mod

x de bf m

y af ce m

= Δ −

= Δ −

gdje je Δ inverz od Δ po modulu m. Dokaz

Poslužit ćemo se Gausovom metodom eliminacije, tj. prvu jednačinu pomnožimo sa d, a drugu sa b. Označimo prvu jednačinu sistema sa I a drugu sa II. Tada je I d II b⋅ − ⋅

( )( )mod

mod

adx bdy de m

cbx dby bf m

+ ≡

+ ≡

a nakon oduzimanja (kako smo rekli) dobijamo

( )( ) ( )( ) ( )( )

( )( )

mod mod mod

mod

adx cbx de bf m x ad cb de bf m x de bf m

x de bf m

⎛ ⎞− ≡ − ⇔ − ≡ − ⇔ Δ = − ⇔⎜ ⎟

⎝ ⎠

⇔ = Δ −

14243

Analogno prethodnom postupku, nakon izračunavanja II a I c⋅ − ⋅ dobijamo

( )( )mody af ce m= Δ − .

Pokažimo još da su gornjim formulama data sva rješenja sistema kongruencija. Neka je ( ),x y proizvoljno rješenje sistema. Tada je ono prethodnog (gornjeg) oblika.

Obrnuto, svaki par ( ),x y gornjeg oblika je rješenje sistema (lahko se dokazuje samim uvršatavanjem). Ovim je teorema dokazana. Primjer 9.1. Riješiti sistem

( )( )

2 3 5 mod 7

3 7 6 mod 7

x y

x y

+ ≡

+ ≡

Page 39: Teorija_brojeva_radna_verzija.pdf

39

Rješenje Prvu jednačinu množimo sa 3, a drugu sa 2, pa ih oduzmemo kako je u dokazu teoreme. Tada dobijamo

( )( )

6 9 15 mod 7

6 14 12 mod 7

x y

x y

+ ≡ ⎫⎪−⎬+ ≡ ⎪⎭a

( )5 3 mod 7y− ≡ Odavde slijedi

( ) ( )5 3 mod 7 4 mod 7y ≡ − ≡

Sada treba odrediti inverz broja 5 po modulu 7, a to je takav broj Δ koji zadovoljava ( )5 1 mod 7Δ = , a takav jedan je 3Δ = .

Sada je rješenje ( )9 mod 7y ≡ − , a to je { }5 7 |y k k∈ + ∈Z , a najmanje pozitivno rješenje je 5y = .

Analogno dobijamo x, Prvu jednačinu množimo sa 7, a drugu sa 3, pa ih oduzmemo kako je u dokazu teoreme. Tada dobijamo

( )( )

14 21 35 mod 7

9 21 18 mod 7

x y

x y

+ ≡ ⎫⎪−⎬+ ≡ ⎪⎭

( )5 17 mod 7x ≡ Odavde slijedi

( ) ( ) ( )5 17 mod 7 10 mod 7 2 mod 7x x≡ ≡ ⇒ ≡ , odnosno { }2 7 |x k k∈ + ∈Z , a najmanje pozitivno rješenje je 2x = . Dakle, rješenje sistema je ( ) ( ){ }, 2 7 ,5 7 |x y k k k∈ + + ∈Z . Jasno je da ih ima beskonačno mnogo! Napomena: Analogno rješevamo sistem od tri linearne kongruencije sa tri nepoznate.

Page 40: Teorija_brojeva_radna_verzija.pdf

40

X TESTOVI DJELJIVOSTI Pomoću kongruencija moguće je dati različite testove djeljivosti, koji su zasnovani na krajnjoj reprezentaciji u različitim bazama (npr. baza 10 i druge baze). Počet ćemo sa testovimo koji koriste decimalnu notaciju (baza 10). Neka je

1 1 0 11 1 0 1 1 0 1 1 0... 10 10 ... 10 10 10 10 ... 10k k k k

k k k k k kn a a a a a a a a a a a a− −− − −= = + + + + = + + + +

gdje su cifre { }0 1, ,..., 0,1,...,9ka a a ∈ . Test djeljivosti stepenima sa bazom 2 Kako je

( ) ( ) ( )10 0 mod 2 10 0 mod 2j j j≡ ⇒ ≡ ∀ ∈N Odavde zaključujemo da je

( )0 mod 2n a≡ , jer

( )

10 1 1 0 0

1 1 11 1 1 1

1 2 11 1

10 10 ... 10

10 10 ... 10 2 5 2 5 ... 2 5

2 2 5 2 5 ... 5

k kk k

k k k k k kk k k k

k k k kk k

n a a a a a a

a a a a a a

a a a

−−

− − −− −

− − −−

− = + + + + − =

= + + + = ⋅ + ⋅ + + ⋅ =

= ⋅ + ⋅ + +

a ovo znači da 02 | n a− , a ovo znači da je ( )0 mod 2n a≡ . Dakle, svaki prirodan broj je kongruentan sa svojom zadnjom cifrom po modulu 2. Jasno slijedi, da je prirodan broj paran ako mu je zadnja cifra parna, tj. ako je zadnja cifra parna tada je i prirodan broj čija je to zadnja cifra paran (blablabla...). Idemo dalje (sve je jasno)

( )

( )

21 0

1 1 0

mod 2

... mod 2 jj j

n a a

n a a a a−

M

pa zaključujemo, prirodan broj n je djeljiv sa 2 j ako je broj 1 1 0...j ja a a a− formiran od njegovih

zadnjih j cifara djeljiv sa 2 j . Test djeljivosti stepenima sa bazom 5 Analogno kao sa 2,

( ) ( ) ( )10 0 mod5 10 0 mod5j j j≡ ⇒ ≡ ∀ ∈N pa sva prethodna priča vrijedi i ovdje. (Da ipak ponovimo) Prirodan broj n je djeljiv sa 5 j ako je broj 1 1 0...j ja a a a− formiran od njegovih zadnjih j cifara djeljiv

sa 5 j . Test djeljivosti brojevima 3 i 9 Kako je

( )10 1 mod 3≡ i ( )10 1 mod 9≡ slijedi da je

( )10 1 modk d≡ gdje je 3d = ili 9d = .

Čemu je 11 1 010 10 ... 10k k

k kn a a a a−−= + + + + kongruentno po modulu d?

Ili drugačije, šta treba oduzeti od broja n da bi bio djeljiv sa d? Da skratimo priču ... ljudi su to davno našli ☺ ( )1

1 1 0 1 1 010 10 ... 10 ... modk kk k k kn a a a a a a a a d−

− −= + + + + ≡ + + + + Dakle, prirodan broj n je djeljiv sa 3 (9) ako mu je zbir cifara djeljiv sa 3 (9).

Page 41: Teorija_brojeva_radna_verzija.pdf

41

Test djeljivosti brojem 11 Zbog ( )( )10 1 mod11≡ − imamo

( ) ( ) ( ) ( )( )( )

111 1 0 1 1 0

0 1 2

10 10 ... 10 1 1 ... 1 mod11

... mod11

k kk kk k k k

k

n a a a a a a a a

a a a a

−−− −= + + + + ≡ − + − + + − + ≡

≡ − + − ±

Dakle, prirodan broj 1 1 0...k kn a a a a−= je djeljiv sa 11 ako je suma njegovih cifara sa parnim indeksima umanjena za sumu njegovih cifara sa neparnim indeksima djeljiva sa 11. Primjer 10.1. Ispitati djeljivost broja 32145687 sa 11. Rješenje Prvo nalazimo sumu brojeva (prema testu) 7+6+4+2=19 8+5+1+3=17 19-17=2 , a 2 nije djeljiv sa 11, pa dati broj nije djeljiv sa 11. Test djeljivosti brojevima 7, 11 i 13 Pošto je 7 11 13=1001⋅ ⋅ , slijedi da je

( ) ( )( )1001 1 mod1000 1000 1 mod1001≡ ⇒ ≡ −

Neka je dat broj 1 1 0...k kn a a a a−= . Ispitajmo kada je on djeljiv sa 1001 u zavisnosti od svojih cifara. Broj n možemo napisati kao (grupišemo po tri cifre)

( ) ( ) ( )( )

11 1 0

0 1 2 3 4 5 6 7 8

3 6 22 1 0 1 2 3 4 5 6 7 8 2 1

2

10 10 ... 1010 100 1000 10 100 1000000 10 100 ...

... 1000...000 10 100 10 10 ... 10

k kk k

kk k k k k k

k

n a a a aa a a a a a a a a

a a a a a a a a a a a a a a a

−−

−− − − −

= + + + + =

= + + + + + + + + +

+ + + = + ⋅ + ⋅ + + ⋅14243

Pošto je ( )( ) ( ) ( )31000 1 mod1001 10 1 mod1001pp≡ − ⇒ ≡ − . Sada ćemo postupiti ovako:

Za 1p = , kongrenciju množimo sa (drugim sabirkom) 3 4 510 100a a a+ + i dobijamo (1) ( ) ( )( )3 4 5 3 4 51000 10 100 10 100 1001a a a a a a+ + ≡ − + + Za 2p = , kongrenciju množimo sa (trećim sabirkom) 6 7 810 100a a a+ + i dobijamo (2) ( ) ( )6

6 7 8 6 7 810 10 100 10 100 1001a a a a a a+ + ≡ + + , i tako nastavimo do 2k − , i vidimo da smo na lijevim stranama kongruencija iscrpili sve sabirke broja n osim prvog, a sada ćemo i njega, pa nakon sabiranja svih kongruencija, dobijamo traženi test. Za prvi sabirak možemo uzeti ovako (ovo vrijedi za svaki broj n) (0) ( )0 1 2 0 1 210 100 10 100 mod 1001a a a a a a+ + ≡ + + Nakon sabiranja dobijamo:

( ) ( )0 1 2 3 4 5 2 110 100 10 100 ... 10 100 mod1001k k kn a a a a a a a a a− −≡ + + − + + + ± + + ili

( )0 1 2 3 4 5 2 1... mod1001k k kn a a a a a a a a a− −≡ − + ±

(Ovdje se naizmjenično mijenja znak, zbog ( )1 p− ). Napomena: Ako broj prikazujemo u bazi b vrijede analogni rezultati (probati dokazati).

Page 42: Teorija_brojeva_radna_verzija.pdf

42

XI WILSONOVA TEOREMA. MALA FERMATOVA TEOREMA J. Wilson (1741-1793), engleski matematičar. Teorema 11.1. (Wilsonova teorema) (Bitna za usmeni ispit) Ako je p prost broj, tada je ( ) ( )( )1 ! 1 modp p− ≡ − .

Napomenimo da je prvi dokazao Wilsonovu teoremu francuski mat. Joseph Louis Lagrange 1770. godine. Prije dokaza, radi ilustracije, uradit ćemo sljedeći primjer i formulisati jednu lemu. Primjer 11.1. Neka je 11p = . ( )11 1 ! 10! 1 2 ... 10− = = ⋅ ⋅ ⋅ Pregrupišimo faktore praveći parove brojeva koji su inverzi po modulu 11.

( )( )( )( )

2 6 1 mod11

3 4 1 mod11

5 9 1 mod11

7 8 1 mod11

⋅ ≡ ⎫⎪

⋅ ≡ ⎪⎬

⋅ ≡ ⎪⎪⋅ ≡ ⎭

Nakon množenja, dobijamo ( ) ( )( )1 2 3 4 5 6 7 8 9 10 1 10 mod11 1 mod11⋅ ⋅ ⋅ ⋅ ⋅ ⋅ ⋅ ⋅ ⋅ ≡ ⋅ ≡ −

Lema 11.1. (nije dokazivana na predavanjima) Za svaki a∈Z i 2 2a p≤ ≤ − , pri čemu je p∈Z prost broj, postoji a ∈Z i 2 2a p≤ ≤ − i a a≠ tako da vrijedi

( )1 moda a p⋅ ≡ . Dokaz

Neka je a∈Z takav da je 2 2a p≤ ≤ − i p∈Z prost broj. Jasno je da za takav broj a postoji inverz po modulu p, jer je p prost broj. Dakle, vrijedi ( )1 moda a p⋅ ≡ . Dokažimo da je 2 2a p≤ ≤ − i da je inverz jedinstven. Posmatrajmo potpun sistem ostataka po modulu p. { }0,1, 2,..., 1p − . Kako je ( ), 1a m = , tada na osnovu teoreme 6.4. slijedi da i skup

( ){ }0, , 2 ,..., 1a a p a− čini potpun sistem ostataka po modulu p. Međutim, kako je

( )0 1 mod p≡/ i

( )1 moda p≡/ (pretpostavimo suprotno, a tada je | 1p a − , a to je nemoguće jer je 1 3a p p− ≤ − < )

i ( ) ( )1 1 modp a p a p− ≡ − ≡/ (dokažimo i ovo)

( ) ( ) ( ) ( ) ( )( ) ( )

1 1 | 1

1 mod

p a p a pa a p a p a p p a p a

p a p a p

− − − = − − + = − ⇒ − − − ⇒

⇒ − ≡ −

Kako je ( )1 1 3p a p≤ − − ≤ − jasno je da ( )| 1p p a− −/ tj. ( )1 modp a p− ≡/ a to znači da vrijedi

( ) ( )1 1 modp a p− ≡/ .

Page 43: Teorija_brojeva_radna_verzija.pdf

43

Sve ovo znači da postoji tačno jedan element y skupa ( ){ }2 ,..., 2a p a− (jer čini potpun sistem

ostataka po modulu p) takav da je ( )1 mody p≡ .

Sada je jasno da je y a a= ⋅ pri čemu je ( ){ }2,3,..., 2a p∈ − . Dokažimo da je a a≠ . Ako je a a= , tada je ( )2 1 moda p≡ , Tada na osnovu teoreme 7.2. slijedi da je | 1p a − ili | 1p a + , a oboje je nemoguće jer je 2 2a p≤ ≤ − . Dakle, a a≠ . Ovim je lema dokazana. Pređimo sada na dokaz Wilsonove teoreme. Dokaz

Razlikujemo dva slučaja: 1) Neka je 2p = . ( ) ( ) ( )( )1 ! 2 1 ! 1! 1 1 mod 2p − = − = = ≡ − Dakle, tvrđenje je tačno za 2p = . 2) Neka je p prost broj i 2p > . Za svaki a, gdje je 2 2a p≤ ≤ − , postoji inverz a po modulu p koji je također u intervalu 2 2a p≤ ≤ − (na osnovu leme 11.1.). Zbog toga ćemo brojeve između 2 i 2p − grupisati u parove (takvih parova ima ( )3 / 2p − ) brojeva koji su jedan drugom inverz po modulu p. Nakon što ih grupišemo, množimo ih, te dobijemo

( ) ( )2 ... 2 1 modp p⋅ ⋅ − ≡ .

Množeći kongruenciju sa ( )1 1p⋅ − dobijamo

( ) ( )1 2 ... 1 1 modp p p⋅ ⋅ ⋅ − ≡ − odnosno, odavde slijedi (jer je ( )( )1 1 modp p− ≡ − )

( ) ( )( )1 ! 1 modp p− ≡ − , što je i trebalo dokazati. Istaknimo da vrijedi i obrat Wilsonove teoreme. Teorema 11.2. (Obrat Wilsonove teoreme) Ako je n∈N takav da je ( ) ( )( )1 ! 1 modn n− ≡ − , tada je n prost broj. Dokaz

Upotrijebit ćemo indirektni dokaz. Pretpostavimo da n nije prost broj, tj. n je složen broj i vrijedi ( ) ( )( )1 ! 1 modn n− ≡ − . Pošto je n složen broj, on se može napisati u obliku n a b= ⋅ , pri čemu je 1 a n< < i 1 b n< < . Zbog 1 a n< < slijedi ( )| 1 !a n − , te iz istog razloga i ( )| 1 !b n −

Kako vrijedi ( ) ( )( )1 ! 1 modn n− ≡ − , slijedi ( )| 1 ! 1n n − + , a kako je n a b= ⋅ , slijedi i

( )| 1 ! 1a n − + .

Međutim, ( )| 1 !a n − i ( )| 1 ! 1a n − + slijedi da a mora dijeliti i njihovu razliku, tj.

( ) ( )| 1 ! 1 1 !a n n− + − − , odavde slijedi |1a odnosno 1a = , a ovo je nemoguće jer je 1 a n< < . Dakle, tvrdnja teoreme je tačna. Vidimo da Wilsonova teorema i njen obrat daju potreban i dovoljan uslov da neki broj bude prost.

Page 44: Teorija_brojeva_radna_verzija.pdf

44

Teorema 11.3. (Mala Fermatova teorema - MFT) Neka je p prost broj i a pozitivan cijeli broj takav da ( )| , 1p a p a⇔ =/ .

Tada je ( )1 1 modpa p− ≡ .

Ovu teoremu dokazao je Euler 1741. godine, a formulisao ju je Fermat 1670. godine, po kome i nosi naziv. Dokaz (razlikuje se od dokaza sa predavanja, ideja je ista)

Posmatrajmo potpun sistem ostataka po modulu p { }0,1, 2,..., 1S p= − . Vidimo da je S p= .

Odredimo osobine skupa { }\ 0S .

Jasno je da vrijedi { }( )\ 0 |x S p x∀ ∈ / .

Dalje, { }( ) ( ), \ 0 modx y S x y x y p∀ ∈ ∧ ≠ ≡/ , tj. nikoja dva (različita) od brojeva 1,2,..., 1p − nisu kongruentna po modulu p. Naime, ako bi bila neka dva (različita) elementa kongruenta po modulu p, tada je ( )( ) ( ) ( ), 1, 2,.., 1 mod 0 mod |j k j k p j k p j k p p j k j k≠ = − ∧ ≡ ⇒ − ≡ ⇒ − ⇒ = a ovo, očigledno, nije moguće. Posmatrajmo skup ( ){ }0, , 2 ,..., 1S a a a p a⋅ = − , vidimo da je Sa p= . Na osnovu teoreme 6.4. skup Sa čini potpun sistem ostataka po modulu p, pa sve prethodno rečeno za { }\ 0S vrijedi i za { }\ 0Sa .

Kako je S Sa= , i kako je { }\ 0Sa podskup potpunog sistema ostataka, znači da za svaki

{ }\ 0x S∈ postoji tačno jedan { }\ 0y Sa∈ tako da je ( )modx y p≡ .

Sada je jasno, da ćemo dobiti 1p − kongruencija tj. za svaki element { }\ 0x S∈ naći ćemo tačno

jedan { }\ 0y Sa∈ tako da je ( )modx y p≡ . Nakon što odredimo sve te kongruencije, pomnožiti ćemo ih međusobno, pa ćemo dobiti

( ) ( )( )2 3 ... 1 1 2 3 ... 1 moda a a p a p p⋅ ⋅ ⋅ ⋅ − ≡ ⋅ ⋅ ⋅ ⋅ − Odavde direktno dobijamo

( ) ( ) ( )1 1 ! 1 ! modpa p p p− ⋅ − ≡ −

Kako je ( )( ), 1 ! 1p p − = , kongruenciju možemo podijeliti sa ( )1 !p − , pa dobijamo

( )1 1 modpa p− ≡ , što je i trebalo dokazati. Posljedica 11.3.1. (Posljedica 1 MFT-e) Ako je p prost broj i a∈N , tada je ( )modpa a p≡ . Dokaz

Moguća su dva slučaja: 1) |p a/ Tada je ( )1 1 modpa p− ≡ , a nakon množenja sa a, dobijamo ( )modpa a p≡ . 2) |p a Tada je ( )0 modpa p≡ i ( )0 moda p≡ pa nakon sabiranja kongruencija slijedi ( )modpa a p≡ .

Page 45: Teorija_brojeva_radna_verzija.pdf

45

Napomena 11.1. Interensatno je napomenuti da je ( )1 1 modpa p− ≡ i za neke brojeve p koji nisu prosti.

( )1203 1 mod121≡ dok je 121 11 11= ⋅ . Navedimo da vrijedi i obrat Male Fermatove teoreme. Teorema 11.4. (nije dokazano na predavanjima) Ako je za svaki a∈N , takav da je ( ), 1a p = vrijedi ( )1 1 modpa p− ≡ , tada je p prost broj. Primjer 11.1. Dokazati, ako je p prost broj tada | 5 2 3 1p pp − ⋅ + . Rješenje Za 2,3,5p = lahko se vidi direktnim provjeravanjem. Neka je 5p > . Jasno je ( ),5 1p = , pa na osnovu posljedice 11.3.1. slijedi ( )5 5 modp p≡ .

Iz istog razloga je i ( )3 3 modp p≡ , pa nakon množenja sa 2, dobijamo

( )3 2 3 2 modp p⋅ ≡ ⋅ Nakon oduzimanja, dobijamo

( )5 2 3 5 2 3 modp p p− ⋅ ≡ − ⋅ , a nakon dodavanja 1, dobijamo

( ) ( )5 2 3 1 5 2 3 1 mod 5 2 3 1 0 modp p p pp p− ⋅ + ≡ − ⋅ + ⇒ − ⋅ + ≡ , što je trebalo dokazati. Primjer 11.2. Odrediti najmanji pozitivni ostatak pri dijeljenju broja 2013 sa 11. Rješenje Prema MFT slijedi

( )11 13 1 mod11− ≡ tj. ( )103 1 mod11≡ . Nakon stepenovanja sa 20, dobijamo

( ) ( )200 200 2003 1 mod11 3 1 mod11≡ ⇒ ≡ Nakon množenja sa 3, dobijamo

( )2013 3 mod11≡ Dakle, traženi ostatak je 3. Primjer 11.3. Odrediti ostatak pri dijeljenju broja 4538 sa 5. Rješenje Prema MFT slijedi

( )5 18 1 mod 5− ≡ tj. ( )48 1 mod 5≡ . Nakon stepenovanja sa 113, dobijamo

( )4528 1 mod 5≡ a nakon množenja sa 8, dobijamo

( )4538 8 3 mod 5≡ ≡ Dakle, ostatak je 3.

Page 46: Teorija_brojeva_radna_verzija.pdf

46

Posljedica 11.3.2. (Posljedica 2 MFT-e) Ako je p prost broj, a∈N i |p a/ , tada je 2pa − inverz od a po modulu p. Dokaz

Po MFT, pošto |p a/ , slijedi ( ) ( )1 21 mod 1 modp pa p a a p− −≡ ⇒ ⋅ ≡

a ovo znači da je 2pa a −= , što je trebalo dokazati.

Page 47: Teorija_brojeva_radna_verzija.pdf

47

XII EULEROVA TEOREMA Mala Fermatova teorema je tvrdila da je ( )1 1 modpa p− ≡ ukoliko je p prost broj i |p a/ , a ona je specijalan slučaj Eulerove teoreme, što ćemo kasnije vidjeti. Da bi smo mogli iskazati Eulerovu teoremu, najprije ćemo definisati Eulerovu funkciju. Definicija 12.1. (Eulerova funkcija) Eulerova funkcija je :ϕ →N N definisana sa ( ) ,n broj prirodnih brojeva manjih ili jednakih od n koji su relativno prosti sa brojem nϕ = .

Primjer 12.1. Naći ( )10ϕ , ( )5ϕ i ( )7ϕ .

( ) { }10 1,3,7,9 4ϕ = =

( ) { }5 1,2,3,4 4ϕ = =

( ) { }7 1,2,3,4,5,6 6ϕ = = Teorema 12.1. Ako je p∈N , 1p > i ( ) 1p pϕ = − , tada je p prost broj. Dokaz

Neka vrijede pretpostavke teoreme. Pretpostavimo da p nije prost, tj. neka je složen. Tada p ima djelilac (jer je složen), recimo a, koji ima osobinu 1 a p< < . Tada a i p nisu relativno prost (jer ih dijeli a). To znači da je ( ) 2p pϕ ≤ − , što je kontradikcija. Teorema 12.2. Neka je p prost broj i a∈N . Tada je ( ) 1a a ap p pϕ −= −

Dokaz (Dokazati za vježbu!)

Posmatrajmo brojeve 11, 2,..., ,..., 2 ,..., ,..., ap p p p p p−⋅ ⋅

Jasno se vidi da su ovdje svi (prirodni) brojevi od 1 do ap , a mi treba da utvrdimo koliko od tih brojeva je relativno prosto sa ap . Od posmatranih brojeva, njih 1ap − je djeljivo sa p, tj. nije relativno prosto sa ap , a svi ostali pri dijeljenju sa p, daju ostatak različit od 0, tj. nisu djeljivi sa p. Na osnovu prethodnog, ukupan broj relativno prostih sa ap jeste kada od posmatranih brojeva (kojih ima ap ) oduzmemo 1ap − (one koji jesu relativno prosti sa ap , tj. djeljivi sa p), tj.

( ) 1a a ap p pϕ −= − , što je i trebalo dokazati. Definicija 12.2. (uopštena definicija) Funkciju :f →N C za koju vrijedi

1) ( )1 1f =

2) ( ) ( ) ( )f mn f m f m= ⋅ za sve m,n takve da je ( ), 1m n = zovemu multiplikativna funkcija.

Page 48: Teorija_brojeva_radna_verzija.pdf

48

Teorema 12.3. Eulerova funkcija je multiplikativna funkcija. Dokaz

Dokaz u knjizi „Elementarna teorija brojeva“ str. 139. Napomena Koristeći teoremu 12.3. možemo relativno brzo za neke „velike“ prirodne brojeve n naći ( )nϕ . Primjer 12.2. ( ) ( ) ( ) ( ) { }40 5 8 5 8 4 1,3,5,7 4 4 16ϕ ϕ ϕ ϕ= ⋅ = ⋅ = ⋅ = ⋅ = .

Naravno, ova formula nije toliko efektna za određivanje ( )nϕ ukoliko je n veliki broj.

Zato ćemo dati jednu efektniju formulu za određivanje ( )nϕ . Teorema 12.4. Ako je 1 2

1 2r

rm p p pα α α= ⋅ ⋅ ⋅K rastav prirodnog broja m na proste faktore, onda vrijedi:

( )1 2

1 1 11 1 1r

m mp p p

ϕ⎛ ⎞⎛ ⎞ ⎛ ⎞

= − − −⎜ ⎟⎜ ⎟ ⎜ ⎟⎝ ⎠⎝ ⎠ ⎝ ⎠

K .

Dokaz

Pošto je 1 21 2

rrm p p pα α α= ⋅ ⋅ ⋅K , gdje su 1 2, ,..., rp p p prosti brojevi, po teorema 12.3. slijedi da je

(1) ( ) ( ) ( ) ( )1 21 2

rrm p p pα α αϕ ϕ ϕ ϕ= ⋅ ⋅ ⋅K .

Na osnovu teoreme 12.2. slijedi da je

( ) ( )1 1 11 1i i i i ii i i i i i

i

p p p p p pp

α α α α αϕ − − ⎛ ⎞= − = − = −⎜ ⎟

⎝ ⎠ (za neki 1,2,...,i r= ).

Iz (1) i prethodnog slijedi:

( ) 1 2

1 2

1 21 2

1 21 2

1 2

1 1 11 1 1

1 1 11 1 1

1 1 11 1 1

r

r

rr

rrm

r

m p p pp p p

p p pp p p

mp p p

α α α

α α α

ϕ⎛ ⎞ ⎛ ⎞ ⎛ ⎞

= − ⋅ − ⋅ ⋅ − =⎜ ⎟ ⎜ ⎟ ⎜ ⎟⎝ ⎠ ⎝ ⎠ ⎝ ⎠

⎛ ⎞⎛ ⎞ ⎛ ⎞= ⋅ ⋅ ⋅ − − ⋅ ⋅ − =⎜ ⎟⎜ ⎟ ⎜ ⎟

⎝ ⎠⎝ ⎠ ⎝ ⎠

⎛ ⎞⎛ ⎞ ⎛ ⎞= − − ⋅ ⋅ −⎜ ⎟⎜ ⎟ ⎜ ⎟

⎝ ⎠⎝ ⎠ ⎝ ⎠

K

K K1442443

K

Primjer 12.3. Naći ( )120ϕ .

3 1 1120 2 3 4 5 2 3 5= ⋅ ⋅ ⋅ = ⋅ ⋅

( ) 1 1 1 1 2 4 24 4120 120 1 1 1 120 8 4 322 3 5 2 3 5 3

ϕ ⋅⎛ ⎞⎛ ⎞⎛ ⎞= − − − = ⋅ ⋅ ⋅ = = ⋅ =⎜ ⎟⎜ ⎟⎜ ⎟⎝ ⎠⎝ ⎠⎝ ⎠

Page 49: Teorija_brojeva_radna_verzija.pdf

49

Napomena (vezano za teoremu 12.3.) Ako je 1 2

1 2k

kn p p pαα α= ⋅ ⋅ ⋅K , gdje su 1 2, ,..., kp p p relativno prosti brojevi, tada je

( ) ( ) ( ) ( )1 21 2 ... k

kn p p pαα αϕ ϕ ϕ ϕ= ⋅ ⋅ ⋅ . Ovo ćemo dokazati metodom matematičke indukcije. Za 1k = , očigledno! Za 2k = , uzimajući da je 1

1m pα= i 22n pα= , a kako su m i n relativno prosti, na osnovu teoreme

12.3. slijedi tvrđenje. Neka tvrdnja vrijedi za neko k, dokažimo da vrijedi i za k+1.

( ) ( ) ( )( ) ( ) ( ) ( )

1 1 11 2 1 2 1 2

11 2

12.3.

1 2 1 1 2 1 1 2 1

1 2 1...

k k k k k k

k k

teorema

k k k k k k

k k

n p p p p p p p p p p p p

p p p p

α α α α α αα α α α α α

α αα α

ϕ ϕ

ϕ ϕ ϕ ϕ

+ + +

+

+ + +

+

= ⋅ ⋅ ⋅ ⋅ = ⋅ ⋅ ⋅ ⋅ ⇒ ⋅ ⋅ ⋅ ⋅ =

= ⋅ ⋅ ⋅ ⋅

K K K

Dakle, tvrdnja vrijedi (na osnovu metode matematičke indukcije). Definicija 12.3. (Redukovani sistem ostataka) Redukovani sistem ostataka po modulu m je skup cijelih brojeva ir sa osobinom da je ( ), 1ir m = i

( )modi jr r m≡ i da za svaki cijeli broj x postoji ir takav da je ( )modix r m≡ . Jedan redukovani sistem ostataka po modulu m je skup svih brojeva { }1, 2,...,a m∈ , takvih da je

( ), 1a m = . Vidi se, a na osnovu definicije Eulerove funkcije, da je broj elemenata redukovanog sistema ostataka po modulu m jednak ( )mϕ . Primjer 12.4. Skup { }1,5,7,11,13,17 je redukovani sistem ostataka po modulu 18.

Za svaki { }1,5,7,11,13,17a∈ vrijedi ( ),18 1a = .

Dalje, nikoja dva elementa iz skupa { }1,5,7,11,13,17 nisu kongruentna po modulu 18.

Uzmimo broj 35. Vidimo da je ( )35,18 1= . Nađimo broj iz skupa { }1,5,7,11,13,17 koji je kongruentan sa 35 po modulu 18. Pri dijeljenju 35 sa 18, ostatak je 17, a pri dijeljenju broja 17 sa 18, ostatak je 17. Dakle ( )35 17 mod18≡ . Teorema 12.4. Neka je ( ){ }1 2, ,..., mr r rϕ redukovani sistem ostataka po modulu m i neka je a∈N i ( ), 1a m = .

Tada je skup ( ){ }1 2, ,..., mar ar arϕ redukovani sistem ostataka po modulu m.

Dokaz

Dokažimo prvo da je svaki element skupa ( ){ }1 2, ,..., mar ar arϕ relativno prost sa m.

Kada bi bilo ( ), 1iar m d= > , tada bi bilo ( ) ( )| | | |id a d m d r d m∧ ∨ ∧ , a odavdje slijedi da je

1d = , jer je ( ), 1a m = i ( ), 1ir m = , dakle nije moguće ( ), 1iar m d= > . Pokažimo da nikoja dva od

brojeva ( ) ( )( ), , 1, 2,...,i jar ar i j i j nϕ≠ ∈ nisu kongruentna po modulu m. Ako bi bilo

( )modi jar ar m≡ tada je, nakon dijeljenja sa brojem a i kako je ( ), 1a m = , ( )modi jr r m≡ , što je kontradikcija.

Page 50: Teorija_brojeva_radna_verzija.pdf

50

Stigli smo do veoma važne teoreme, koja se vrlo često koristi u rješavanju zadataka, a to je Eulerova teorema. Teorema 12.5. (Eulerova teorema) Ako su m∈N i a∈Z takvi da je ( ), 1a m = .

Tada je ( ) ( )1 modma mϕ ≡ Dokaz

Neka je ( ){ }1 2, ,..., mr r rϕ redukovani sistem ostataka po modulu m. Kako je ( ), 1a m = , tada je

prema prethodnoj i skup ( ){ }1 2, ,..., mar ar arϕ redukovani sistem ostataka po modulu m.

Za svaki od brojeva skupa ( ){ }1 2, ,..., mar ar arϕ postoji tačno jedan element skupa ( ){ }1 2, ,..., mr r rϕ ,

tako da vrijedi ( ) ( )( )mod , 1, 2,...,i jar r m i j mϕ≡ = .

Dakle, postoje brojevi ( ) ( ){ }1 2, ,..., 1, 2,...,mi i i mϕ ϕ∈ takvi da vrijedi

( )( )

( ) ( )( )

1

2

1

2

mod

mod

modm

i

i

im

ar r m

ar r m

ar r mϕϕ

M

Množenjem ovih kongruencija, dobijamo:

( ) ( )( ) ( )

( ) ( ) ( )1 21 2 1 2 1 2... ... mod ... ... mod

m

mi i im m mar ar ar r r r m a r r r r r r m

ϕ

ϕϕ ϕ ϕ⋅ ⋅ ⋅ ≡ ⋅ ⋅ ⋅ ⇔ ⋅ ⋅ ⋅ ⋅ ≡ ⋅ ⋅ ⋅

Kako je svaki od brojeva ( )1 2, ,..., mr r rϕ relativno prost sa brojem m, to na osnovu teorema 6.2.

možemo izvršiti skraćivanje, pa je ( ) ( )1 modma mϕ ≡ Nalaženje inverza broja a po modulu m pomoću Eulerove teoreme Pomoću Eulerove teoreme možemo odrediti inverz broja a po modulu m. Kako je ( ), 1a m = te kako je ( ) ( ) ( ) ( )11 mod 1 modm ma m a a mϕ ϕ −≡ ⇔ ⋅ ≡ , slijedi da je ( ) 1maϕ − inverz broja a po modulu m. Primjer 12.4. (Mala Fermatova teorema – kao posljedica Eulerove teoreme) Ako je m prost broj dobijamo Malu Fermatovu teoremu. Neka je ( ), 1a m = i neka je m prost broj. Tada je ( ) 1m mϕ = − , tada je prema Eulerovoj teoremi

( ) ( )1 modma mϕ ≡ , odnosno, ( )1 1 modma m− ≡ .

Page 51: Teorija_brojeva_radna_verzija.pdf

51

Primjer 12.5. Naći ostatak pri dijeljenju

5103 sa 35. Rješenje Kako je ( )3,35 1= , na osnovu Eulerove teroeme, slijedi

( ) ( )353 1 mod35ϕ ≡

( ) ( ) ( )35 7 5 6 4 24ϕ ϕ ϕ= ⋅ = ⋅ = 510 100000 24 4166 16= = ⋅ + , odavde slijedi

( ) ( )5 416610 24 163 3 3 mod35≡ ⋅ , a kako je ( )243 1 mod 35≡ , dalje slijedi

( )510 163 3 mod35≡ .

( ) ( )4 16 43 11 mod 35 3 11 mod 35≡ ⇒ ≡

( ) ( )2 4 211 16 mod 35 11 16 mod 35≡ ⇒ ≡

( )216 11 mod 35≡ . Dakle, ostatak je 11. Primjer 12.6. Naći ostatak pri dijeljenju 259317 sa 15. Rješenje

( ) ( )259 259317 2 mod15 317 2 mod15≡ ⇒ ≡

Kako je ( )2,15 1= , na osnovu Eulerove teoreme slijedi ( ) ( )152 1 mod15ϕ ≡

( ) ( ) ( )15 5 3 4 2 8ϕ ϕ ϕ= ⋅ = ⋅ =

( )82 1 mod15≡

( ) ( ) ( )32 32259 8 3 259 8 3259 8 32 3 2 2 2 2 2 2 mod 15= ⋅ + ⇒ = ⋅ ⇒ ≡ ⋅

Odavde slijedi, zbog ( )82 1 mod15≡ ,

( )259 32 2 mod 15≡ , odnosno, ostatak je 8.

Page 52: Teorija_brojeva_radna_verzija.pdf

52

XIII PRIMITIVNI KORIJEN Na osnovu Eulerove teoreme, ako je m∈N i a∈Z i ( ), 1a m = , tada je ( ) ( )1 modma mϕ ≡ . Dakle, postoji bar jedan x∈N koji pri uslovima Eulerove teoreme zadovoljava kongruenciju

( )1 modxa m≡ . Kako je skup prirodnih brojeva dobro uređen skup, postoji najmanji broj x∈N ,

takav da je ( )1 modxa m≡ . Definicija 13.1. (Red broja a po modulu m) Neka su a i m relativno prosti prirodni brojevi. Najmanji broj x∈N , takav da je ( )1 modxa m≡ zovemo red broja a po modulu m, u oznaci mord a . Primjer 13.1.

7 2ord Jasno je, da je ( )32 1 mod 7≡ , i to ja najmanji broj. Dakle 7 2 3ord = . Za nalažanja reda broja a po modulu m, koristimo sljedeću teoremu. Teorema 13.1. Neka su a∈Z i m∈N , tako da je ( ), 1a m = . Broj x je rješenje kongruencije ( )1 modxa m≡ akko je |mord a x . Dokaz

„⇐“ Ako je |mord a x , tada postoji k ∈Z , takav da je mx k ord a= ⋅ . Odavde slijedi

( )mkord axa a≡ .

Kako je ( )1 modmord aa m≡ , slijedi ( ) ( )1 modmkord ax ka a m≡ ≡ . Dakle ( )1 modxa m≡ .

„⇒ “ Neka je ( )1 modxa m≡ . Na osnovu definicije 13.1. je mord a x≤ . Za mord a i x na osnovu algoritma dijeljenja postoje (prirodni) brojevi q i r, takvi da je

( )0m mx q ord a r r ord a= ⋅ + ≤ < .

Tada imamo, ( )mkord ax ra a a= ⋅ , a odavde slijedi ( ) ( )modm

kord ax ra a a m≡ ⋅ .

Kako je ( )1 modmord aa m≡ , slijedi ( )modx ra a m≡ , a kako je ( )1 modxa m≡ slijedi

( )1 modra m≡ . Kako je 0 mr ord a≤ < i mord a najmanji broj za kojeg vrijedi ( )1 modmord aa m≡ , slijedi da je 0r = . Sada je |m mx q ord a ord a x= ⋅ ⇒ . Posljedica 13.1.1. Neka su a∈Z i m∈N , tako da je ( ), 1a m = . Tada je ( )|mord a mϕ .

Kako je ( ) ( )1 modma mϕ ≡ , na osnovu prethodne teoreme slijedi ( )|mord a mϕ . Primjer 13.2. Odredimo 17 5ord . Dakle, treba odrediti najmanji prirodni broj x, takav da je ( )5 1 mod17x ≡ .

( )17 16ϕ = . Na osnovu posljedice, slijedi je |16x , dakle { }1, 2, 4,8,16x∈ . Sada direktnom provjerom nalazimo traženo x, pa slijedi da je 17 5 16x ord= = .

Page 53: Teorija_brojeva_radna_verzija.pdf

53

Teorema 13.2. Neka su a∈Z i m∈N , tako da je ( ), 1a m = . Tada je ( ) ( )mod ,i ja a m i j≡ ∈N akko je

( )mod mi j ord a≡ . Dokaz

„⇐“ Neka je ( )mod mi j ord a≡ . Tada postoji k ∈Z , takav da je

m mi j k ord a i j k ord a− = ⋅ ⇒ = + ⋅ . Sada imamo

( ){ ( ) ( )1 mod

mod modm

k

ord ai j i j

m

a a a m a a m≡

⎛ ⎞≡ ⋅ ⇒ ≡⎜ ⎟⎜ ⎟⎝ ⎠

.

„⇒ “ Neka je ( )modi ja a m≡ . Možemo napisati ( )i j i j= + − . Sada je

( )modi j i ja a a m−≡ ⋅ , te kako je ( )modi ja a m≡ slijedi ( )modj i j ja a a m−⋅ ≡ . Kako je

( ) ( ), 1 , 1ja m a m= ⇒ = , podijeliti ćemo kongruenciju ( )modj i j ja a a m−⋅ ≡ sa ja , pa dobijamo

( )1 modi ja m− ≡ . Na osnovu teoreme 13.1. slijedi |mord a i j− a ovo znači da je

( )mod mi j ord a≡ Definicija 13.2. (primitivni korijen po modulu m) Ako je red broja a po modulu m jednak ( )mϕ , broj a se naziva primitivni korijen po modulu m. Dakle, ako je ( )mord a mϕ= , broj a je primitivni korijen po modulu m. Primjer 13.3.

( ) ( )7 73 3 1 mod 7 3 6 7xord x ord ϕ⇒ ≡ ⇒ = = =

( ) ( )17 175 5 1 mod17 5 16 17xord x ord ϕ⇒ ≡ ⇒ = = = pa je 3 odnosno 5 primitivni korijen po modulu 7 odnosno modulu 17. Nemaju svi brojevi primitivne korijene! Na primjer, 81 1ord = , 83 2ord = , 85 2ord = , 8 7 2ord = dok je ( )8 4ϕ = . Dakle, nijedan broj po modulu 8, nema primitivni korijen. Primitivni korijen po datom modulu nije jedinstven! Na primjer, 7 3 6ord = i 7 5 6ord = i ( )7 6ϕ = . Dakle, i 3 i 5 su primitivni korijeni po modulu 7. Teorema 13.3. Ako je r primitivni korijen po modulu m, tada brojevi ( )1 2, ,..., mr r rϕ čine redukovani sistem ostataka. Dokaz

Prvo ćemo dokazati da je svaki od brojeva ( )1 2, ,..., mr r rϕ relativno prost sa m.

Kako je ( ) ( ) ( ){ }( ), 1 , 1 1,2,...,ir m r m i mϕ= ⇒ = ∀ ∈

Dokažimo sada, da nikoja dva od brojeva ( )1 2, ,..., mr r rϕ nisu kongruentna po modulu m, tj. da

( ) ( ){ }( )mod , 1,2,...,i jr r m i j m i jϕ≡ ∀ ∈ ∧ ≠ .

Page 54: Teorija_brojeva_radna_verzija.pdf

54

Ako bi bilo ( )modi jr r m≡ , na osnovu teorema 13.2. i ( ) mm ord rϕ = slijedi da je

( )( )modi j mϕ≡ . Odavde slijedi da je ( ) |m i jϕ − , a kako je ( )i j mϕ− < slijedi da je 0i j i j− = ⇒ = , što je kontradikcija (s pretpostavkom i j≠ ).

Teorema 13.4.

Ako je mord a t= i n∈N , tada je ( ) ( ),n

mtord a

t n= .

Dokaz Neka je ( )n

ms ord a= i ( ),v t n= .

Tada postoji 1 1,t n ∈N , tako da je 1t t v= i 1n n v= . Tada je jasno da je ( )1 11 , ,t n t nv v

⎛ ⎞= =⎜ ⎟⎝ ⎠

.

Dalje slijedi da je 1 11 1

t nv tn ntt n

= = ⇒ = , pa je sada ( ) ( ) ( )11 1 1mod modntn nt ntta a m a a m≡ ⇒ ≡ .

Kako je ( )1 modta m≡ , slijedi ( )1 1 modnta m≡ odnosno ( ) ( )1 1 modtna m≡ .

Na osnovu teoreme 13.1. dobijamo ( ) 1|nms ord a t= .

S druge strane imamo da je ( ) ( ) ( )1 mod 1 modsn nsa m a m≡ ⇒ ≡ , pa na osnovu teoreme 13.1.

slijedi da je ( )1 1, 1

1 1 1 1 1| | | |t n

mt ord a ns t v n vs t n s t s=

= ⇒ ⇒ ⇒ . Dakle, dobili smo da je 1 |t s i 1|s t , odakle slijedi samo 1t s= ± , a kako je 1 1,t s t s∈ ⇒ =N .

Dakle, ( ) ( ) ( ) ( )1 ,,

n nm m

tt t v t sv t ord a t n ord at n

= ⇔ = ⇔ = ⋅ ⇒ = .

Posljedica 13.4.1. Neka je r primitivni korijen po modulu m i neka je n∈N . Tada je nr primitivni korijen po modulu m akko je ( )( ), 1n mϕ = . Dokaz

Iz teoreme 13.4. slijedi da je

( ) ( )( )( )( ) ( ) ( ) ( )( ), 1

, ,n nm

m mm

mord rord r ord r m n mn ord r n m

ϕϕ ϕ

ϕ= = ⇒ = ⇔ =

Teorema 13.5. Ako m∈N ima primitivni korijen, tada ima tačno ( )( )mϕ ϕ primitivnih korijena nekongruentnih po modulu m. Dokaz

Neka je r primitivni korijen po modulu m. Tada na osnovu teoreme 13.3. slijedi da brojevi ( )1 2, ,..., mr r rϕ čine redukovani sistem ostataka. Od svih brojeva ( )1 2, ,..., mr r rϕ , primitivni korijeni

po modulu m jesu samo oni ( ){ }( )1,2,...,nr n nϕ∈ za koje vrijedi (prema posljedica 13.4.1.) da je

( )( ), 1n mϕ = . Brojeva n koji su relativno prosti sa ( )mϕ , odnosno primitivnih korijena nr po

modulu m, ima tačno ( )( )mϕ ϕ , a kako svi primitivni korijeni po modulu m pripadaju

redukovanom sistemu ostataka ( ){ }1 2, ,..., mr r rϕ , to su oni međusobno nekongruentni.

Page 55: Teorija_brojeva_radna_verzija.pdf

55

Primjer 13.4. Neka je 11m = . Odrediti primitivne korijene po modulu m, i broj nekongruentnih. Rješenje Dakle, red nekog broja a po modulu 11 mora biti ( )11 10ϕ = , za kojeg vrijedi ( ),11 1a = i

( )10 1 mod11a ≡ . Nije zgodno nalazati ovakve brojeve (direktno uvrštavamo), a u ovom slučaju je 2a = . Dakle, 2 je primitivni korijen po modulu 11. Kako 11 ima primitivni korijen, slijedi da

postoji ( )( ) ( )11 10 4ϕ ϕ ϕ= = nekongruentna primitivna korijena po modulu 11, a oni se nalaze u

skupu { }2 3 4 5 6 7 8 9 102, 2 , 2 ,2 ,2 ,2 ,2 ,2 ,2 ,2 za koje je eksponent u skupu { }1, 2,3, 4,5,6,7,8,9,10 koji

mora biti redukovani sistem ostataka po modulu 10, a to je skup eksponenata { }1,3,7,9 . Dakle, primitivni korijeni po modulu 11 su:

( ) ( )3 7 92, 2 8, 2 128 7 mod11 , 2 512 6 mod11= = ≡ = ≡ . odnosno, 2, 6, 7, 8 su primitivni korijeni po modulu 11.

Page 56: Teorija_brojeva_radna_verzija.pdf

56

XIV LAGRANGEOVA TEOREMA Posmatrat ćemo polinomijalne kongruencije. Neka je ( ) 1

1 0...n nn nf x c x c x c−

−= + + + polinom sa cjelobrojnim koeficijentima.

Cijeli broj c je korijen (nula) od ( )( )modf x m ako je ( ) ( )0 modf c m≡ .

Primjetimo, ako je ( ) ( )0 modf c m≡ tada je i ( ) ( )1 0 modf c m≡ za svako ( )1 modc c m≡ . Primjer 14.1. Polinom ( ) 2 1f x x x= + + ima tačno dva nekongruentna korijena po modulu 7, i to ( )2 mod 7x ≡ i

( )4 mod 7x ≡ . Primjer 14.2. Ako je p prost broj, onda ( ) 1 1pf x x −= − ima tačno 1p − nekongruentnih korijena po modulu p;

naime, ( )1, 2,..., 1 modx p p≡ − . Teorema 14.1. (Lagrange) Neka je ( ) 1

1 1 0...n nn nf x a x a x a x a−

−= + + + polinom s cjelobrojnim koeficijentima, 1n ≥ , p prost

broj i | np a/ . Tada ( )( )modf x p ima najviše n nekongruentnih korijena po modulu p. Dokaz

Neka je p prost broj i | np a/ . Za dokaz ćemo koristiti matematičku indukciju. Za 1n = imamo ( ) 1 0f x a x a= + .

Korijen od ( )( )modf x p je rješenje linearne kongrunecije u jednoj varijabli

(*) ( )1 0 moda x a p≡ − .

Kako je p prost broj, to je ( )1, 1a p = , pa na osnovu teoreme 7.1. (*) ima tačno jedno rješenje tj.

polinomijalna kongruencija ( )( )modf x p ima tačno jedno rješenje. Dakle, tvrdnja je tačna za 1n = . Prepostavimo da je tvrdnja tačna za polinome stepena 1n − pa dokažimo da je tačna za polinome stepena n. Upotrijebit ćemo indirektni dokaz, tj. pretpostavimo da ( )( )modf x p ima 1n + (ili više) nekongruentnih korijena po modulu p. Neka su to brojevi 0 1, ,..., nc c c (kojih ima 1n + , jer indeks ide od 0 do n). Dakle, { }( ) ( ) ( )0,1,..., 0 modkk n f c p∀ ∈ ≡ Međutim, ( ) 1

1 1 0...n nn nf x a x a x a x a−

−= + + +

( ) 1

0 0 1 0 1 0 0...nn

n nf c a c a c a c a−

−= + + +

Page 57: Teorija_brojeva_radna_verzija.pdf

57

Sada imamo ( ) ( ) ( ) ( ) ( )( )( ) ( )( ) ( )

( ) ( ) ( )

1 10 0 1 0 1 0

1 2 2 1 2 3 3 20 0 0 0 1 0 0 0 0 1 0

1 2 2 1 2 3 3 20 0 0 0 1 0 0 0 1

:

...

... ... ...

... ...

n n n nn n

n n n n n n n nn n

n n n n n n n nn n

g

f x f c a x c a x c a x c

a x c x x c xc c a x c x x c xc c a x c

x c a x x c xc c a x x c xc c a

− −−

− − − − − − − −−

− − − − − − − −−

=

− = − + − + + − =

= − + + + + + − + + + + + + − =

= − + + + + + + + + + +

( )

( ) ( )0

x

x c g x⎧ ⎫⎪ ⎪ = −⎨ ⎬⎪ ⎪⎩ ⎭1444444444444442444444444444443

Odavde je jasno da je ( )( )deg 1g x n= − tj. da je polinom ( )g x stenena 1n − .

Koeficijent uz 1nx − polinoma ( )g x je na i kako | np a/ , prema pretpostavci (pretpostavka

matematičke indukcije) ( )( )modg x p ima najviše 1n − nekongruentnih korijena po modulu p. S druge strane, ako su 0 1, ,..., nc c c korijeni od ( )( )modf x p tada su 1,..., nc c korijeni od

( )( )modg x p . Pokažimo to:

{ }( ) ( ) ( ) ( )00,1,..., 0 modkk n f c f c p∀ ∈ ≡ ≡ Odavde je

( ) ( ) ( ) ( ) ( ) ( )0 00 mod 0 modk k kf c f c p c c g c p− ≡ ⇒ − ≡ za svako { }0,1,...,k n∈ . Kako je { }( ) ( ) ( )0 01,..., mod |k kk n c c p p c c∀ ∈ ≡ ⇒ −/ /

Odavde jasno slijedi da je 0 kc c≠ jer u suprotnom bi bilo ( )0| kp c c− a ovo znači ( )0 modkc c p≡ a ovo nije moguće! Kako smo utvrdili da je 0 kc c≠ i ( ) ( ) ( )0 0 modk kc c g c p− ≡ slijedi da je

{ }( ) ( ) ( )1,..., 0 modkk n g c p∀ ∈ ≡ Ova zadnjo znači da polinomijalna kongruencija ( )( )modg x p ima n nekongruentnih korijena. Iz pretpostavke da ( )( )modf x p ima 1n + (ili više) nekongruentnih korijena po modulu p slijedi

da ( )( )modg x p takav da je ( )( )deg 1g x n= − , ima istovremeno i 1n − i n nekogruentnih korijena po modulu p, a to je kontradikcija. Dakle, ( )( )modf x p , ( )( )deg f x n= , ima najviše n nekongruentnih korijena po modulu p.

Page 58: Teorija_brojeva_radna_verzija.pdf

58

Teorema 14.2. (Posljedica Lagrange-ove teoreme) Ako je d prirodan broj, p prost broj i | 1d p − , tada polinomijalna kongruencija ( )( )1 moddx p− ima tačno d nekongruentnih korijena po modulu p. Dokaz

Neka je p prost broj. Kako ( )| 1 1d p e de p− ⇒ ∃ ∈ = −Z . Dakle, 1p d e− = ⋅ (*) Tada je

( ) ( ) ( ) ( )( ) ( ) ( )1 21 1 1 1 1 ... 1 1e d e d ep d e d e d dx x x x x x x g x− −− ⋅− = − = − = − + + + = −

gdje je ( ) ( ) ( )1 2 ... 1d e d eg x x x− −= + + + .

Po MFT, kongruencija ( ) ( )1 11 0 mod 1 modp px p x p− −− ≡ ⇔ ≡ je zadovoljena ako |p x/ , a to će

biti zadovoljeno ako je 0x x p≠ ∧ < , odnosno ako je { }1, 2,..., 1x p∈ − . Kako x pripada podskupu potpunog sistema ostaka po modulu p, znači da su sva rješenja kongruencije ( )1 1 modpx p− ≡ , nekongruentna po modulu p, i ima ih tačno { }1,2,..., 1 1p p− = − . Očigledno, svaki korijen od ( )( )1 1 modpx p− − je korijen od ( )( )1 moddx p− ili od ( )( )modg x p .

Tada, prema Lagrange-ovoj teoremi ( )( )modg x p ima najviše ( )1d e − nekongruentnih korijena po modulu p. Odnosno, iz (*) slijedi ( )1 1d e de d p d− = − = − −

Isto je očigledno, da svaki korijen od ( )( )1 1 modpx p− − koji nije korijen od ( )( )modg x p mora

biti korijen od ( )( )1 moddx p− .

Broj takvih korijena za ( )( )1 moddx p− je najmanje ( ) ( )1 1p p d d− − − − = (i sva su ona nekogruentna po modulu p). S druge strane, prema Lagrange-ovoj teoremi, ( )( )1 moddx p− ima najviše d nekongruentnih

korijena po modulu p, pa prema tome i prethodno rečenom, slijedi da ( )( )1 moddx p− ima tačno d korijena nekongruentnih po modulu p, što je i trebalo dokazati.

Page 59: Teorija_brojeva_radna_verzija.pdf

59

Dodatak: pisemni dijelovi ispita 14.2.2005 Grupa I

1. Dokazati da je 2kn = ako je 2 1n + prost broj. 2. Odrediti primitivne korijene po modulu 19.

3. Zmaj ima 1987 glava, vitez može jednim potezom mača odsjeći 1, 17, 21 ili 33 glave, ali pri

tome zmaju naraste 10, 14, 6 ili 48 glava redom. Može li vitez ubiti zmaja? (Zmaj je ubijen ako mu vitez odsiječe sve glave.)

4. Student za 19 dana riješi 73 zadatka, za prvih 11 dana rješava po isti broj zadataka x, a

preostalo dana također po isti broj zadatka y. Naći x i y.

5. Riješiti jednačinu 10 10xy x y= + . Grupa II

1. Ako su a i b prirodni brojevi, dokazati da je ( ) ( ) [ ], , nnn nn a b a b a b∀ ∈ + ≤ +N . 2. Riješiti linearnu kongruenciju ( )7 12 mod17x ≡ na više različitih načina (maksimalno 4).

3. Omer duguje Ahmedu 130 KM, pri čemu Omer posjeduje novčanice isključivo od 20 KM, a

Ahmed novčanice od 50 KM. Na koji način će Omer vratiti dug.

4. Pet duži je konstruisano iz tačke A. Zatim je iz nekih od slobodnih krajeva konstruisano pet novih duži i tako nekoliko puta. Na kraju, prebrojano je 700 slobodnih krajeva. Da li je dobro brojano?

5. Riješiti jednačinu 2 2 3 11x xy x y− + − = .

9.4.2005

1. a) Dokazati ( ) ( ) ( ), , 1 , 1m nm n a b a b∀ ∈ = ⇒ =N b) Odrediti najmanji prirodni broj n koji pri dijeljenju sa 4, 6, 8, 10 i 12 daje redom ostatke 2,

4, 6, 8 i 10. 2. Odrediti broj a ako je ( )2a aτ= ⋅ . 3. Dokazati ( ) ( )2, 4,6,..., 2 ,... 20 16 3 1 0 mod 323n n nn k∀ = + − − ≡ .

4. U sobi su stolice sa 3 i 4 noge. Kada na sve stolice sjednu ljudi u sobi je ukupno 69 nogu.

Koliko u sobi ima stolica svake vrste?

5. Riješiti u skupu Ν jednačinu 3 2 3 0x x x+ + − = .